You are on page 1of 453
Engineering Mathematics For GATE 2019 and ESE 2019 Prelims Comprehensive Theory with Solved Examples Including Previous Solved Questions of GATE (2003-2018) and ESE-Prelims (2017-2018) Note: Syllabus of ESE Mains Electrical Engineering also covered MADE ERSY i (ees MADE EASY Publications Corporate Otce: 44:04, Kalu Sarai (Near Hauz Khas Metro Station), New Delhi-1 10016 smal: infomep@madecasy.in Contact: 011-45124660, 8860378007 Visit us at: wwwcmadeeasypublicatons org Engineering Mathematics for GATE 2019 and ESE 2019 Prelims © Copyright, by MADE EASY Publications. All rights are reserved. No part of this publication may be reproduced, stored in or introduced into a retrieval system, or transmitted in any form or by any means (electronic, mechanical, photo-copying, recording or otherwise), without the prior written permission of the above mentioned publisher of this book, st Edition 2009 2nd Edition —: 2010 3rd Edition 201 “th Falition 2012 Sth Edition 2013 6th Editon 2014 7th Edition 2015 ‘th Editon 2016 ‘9th Edition 2017 ‘10th Edition: 2018 MADE EASY PUBLICATIONS has taken due care in collecting the data and providing the solutions, before Publishing this book, Inspite of this, if any inaccuracy or printing etror occurs then MADE EASY PUBLICATIONS. ‘owes no responsibilty. We will be grateful if you could point out any such ent. Your suggestions wil be appreciated, Pi led at : Fine Offset Printers, Tronica City, (UP) Preface (Over the period of time the GATE and ESE examination have become more challenging due to increasing number of candidates. Though every candidate has ability to succeed but competitive environment, in-depth knowledge, quality guidance and good source of study is required to achieve high level goals. B, Singh (Ex.(E5) The new edition of Engineering Mathematics for GATE 2019 and ESE 2019 Prelims has been fully revised, updated and edited. The whole book has been divided into topicwise sections | have true desire to serve student community by way of providing good source of study and quality guidance. | hope this book will be proved an important tool to succeed in GATE and ESE ‘examination. Any suggestions from the readers for the improvement of this book are most welcome. B. Singh (Ex. IES) Chairman and Managing Director MADE EASY Group iit SYLLABUS GATE and ESE Prelims: Civil Engineering Linear Algebra: Nai: alge; Systems finest equations; Eigen values ad igen vector, Calculus: Functions of single varible; Unt, continuity and diferentbity; ‘Mean valve theorems, local maxima and minima, Taylor and Matta seg, Evaluation of definite and indent meals, appicaton of deft neg ty bain rea ad volume; Partial deviates lol derivate, Gradient, Deen and Gal, Vector idenites, Detar derivatives, Lin, Surlace and Voone Integial Soles, Gaus and Greens theorems, Oralnary Differential Equation (ODE) Fist order {near and nonlnest) uations higher onde Hina equations wth constant ceticents Euler Cauchy equatonsLaplace wansform andsappation in slving Inst ODEs tl end boundary value problems Partial Differential Equation (PDE}: Fur series sertion of vances; soltions of one-imensonal din eqiion; fist and second oer ote Alesina vave equation ae two- densa ple equation, Probability and Statistics: Defntons of probability and sampling theorems: Condonal probabity, Discrete Radom vanables: Posson and Sintnal tstutions; Continuous rendorn vibes normal andexpanentl Sstbunine Descriptive statist - ean, median, md and standard deviation Hypo, testing Numerical Methods: Accuracy andprecio; ero analysis Mum solitons of lear ond non-linear algebra equations; Least sqiae approximation, Heswons ond Lagrange polyaomias sumer diferent, integration by trapesdal and Simpsons rule, singe and moli-step methods fr fat nde, ‘ferential equations GATE and ESE Prelims: Mechanical Engineering Linear Algebra: Waicsigedra, systems of near Cautions, eigenvalues ant lgenvectes Calculus: Functions of single varabl, mt, continuity end feted, ‘mean valve theorems, ndetesminate farms evaluation of define and mpopy Integas double and tpl integrals partaldewatives, ata crv Series none and two variables), maxima and minima, fouser sees: grader, vergence and cr, vector idee, dretonal dertative, hie sunace snd valumeintegak, applications af Gaus, Stokes nd Gren’ theorems Differential equations: Fst order equations (linear and nantnes); higher order neat leretial equations wth constant coins Euler-Cant ‘boundary vale poems; Laplace tansformsslutens of heat, wate and Laplace’ equations Complex Variables: Analytic functions Cauchy-Riesann equations Cauchy's integral theorem apd incegtal form olor and Laurent eres Probability and Statistics: Defitions of probability, sampling theo, ‘ondtonl probity: mean, median, made and standord deviation, random ‘arabes, binomial Poison and normal dstibutens, Numerical Methods: Numeral solutions af linear and nontines algebioc uations integration by trapezoidal and Smpeon’ les, single nd mulesey methods fo cifererta equations ‘GATE and ESE Prelims: Electrical Engineering LUnear Alger Has Alga, Swomsoinsregsins gens Eger Calculus: Mean valve theorems, Theorems af itera calls, vlan of definite and improper integial, Fata Deriatwes, Maxima and minima, Moto inegals, Four ses, Vea denies, Dvectionl deivates, Lig near, Surface integral, Volume ing Soke theorem, Gauss thee, Greets theorem Differential equations: Fist order equations flnear and nolinea, Higher der inearcfrenal equations with constant coafcens Method of vanaion of parameters, Cauchy's equation, Ful eto, Intl and bovadary sale Dablems, Partial Dire Equations, Method of separation af vaste, Complex Variables: Anaytic functions, Cauchy intgil theo, Caxchys neat! ormul Tayo series, Laurent sees, Residue theoern, Slut inter Probability and Statistics: Sampling theres, Conditional probsbiny, Mean, Medio. Made, Sendard Deviation fandom valle, Dict and Contnoon Aistristions, Fosson distribution, Norma dstbution, Binomial dstbunen {onelaton analysts, Regression aay Numerical Methods: Solutions of nonlinear algebra equations Singh and ‘Mult-step methods deena equations. Transform Theoty: Four Kansform, Laplace Tansform,2anstx Electrical Engineering ESE Mains ‘aut theory, ign values & Eigen vects, sytem of linear equations, Numerical methods for solution af non-linkaalgebiaeeouations aed ferential equations, itera) calculus, partal devas, maxima and ‘minima, Line, Sure and Volume Intel, Four sees, nea, nonineo and partial iferetal equations, inital and. boundary value, problems complex vaabes, rs apd Laurent’ series, residue theorem, probably and statistics indamental, Sampling theorem, ado vials, mal ang Poisson aston, catelaton an regtesson analy GATE and ESE Prelims: Electronics Engineering Lear Algebra: Yer space, bass, linear dependence and independence ‘matric algebra, tigen valves and eigen vectors rank Slaton of hese ations ~ existence and uniqueness Calculus: Mean vale theorems, theorems of integral cles, evaluation of definite and improper intra, parti dereaties maxims and minancy ‘ltl integra ne, surface and volume integrals Taylor snes I Differential equations: ist oder equations (het and antes), higher ter linear diferent equations, Cauchy's and Furs equations, methods of solution using veratan of patameters,complementay funn sh a sor inveoral aril diferent equations, viable separable method, iil and ‘bounday valve probes ' Vector Analysis: Yes tn plane and space, vector operations, gradient, Aivergnce ad eu, Gauss, Greens and Stoke’ theorems Complex Analysis: Analytic functions, Covey’ integral tearm, Cauchy's integra frmat: aor and Laurent eros, residue theme, Numerical Methods:Salitin of nonlinear equotions single and multistep ‘methods fr aiferenal equations convergence citer Probability and Statistics: Mean, median, mode and standae destin; combinatorial probity, probabity dstibution functions =. binomial Poisson, exonentl and normal Joint and condtonal probably, Creation and regression ana neers GATE: Instrumentation Engineering Linear Algebra: atic aleta, ystems olin equations igen vies anc Calculus + Mean save theorems, theres of integral cles, pata erates, maxima ad minina, multiple integrals, Fourier sere, vector ents, ‘ine, surface and volume Integral, Stokes, Gauss and Crees theorems. Differential Equations Fst over equation (ea nc nota) higher one ner ferential eaunions with constant exes, method of varatlon of orameters, Cael’ and Euler equaon, ld boundary vue pole, ‘alton of parts femal eqatonsvariablesepaabe eth, Anaisis of complex variables: : Analytic functions, Cauchy's Ineoral theorem and inte mula, Tylana Louies series, side theorem, solution of integrals, ‘Complex Variables: Anji funtions Caucty’sntegal theorem an intial forma, aylor’ a Lauren seis, Rese theory sotion neal Probability and Statistics: Sampling theres, conditional probably, mean, medlan, mode ad standard deviation, andor variables, dscrete and Continuous estabutons normal Plsson and binmldstibutons Numerical Methods : hse inversion, solutions of noone algebraic ‘equations, iterative metiods for soling alfereaal equations, numerical Integration, regtesson and coelaion analysis GATE: Computer Sclence &T Engineering Linear Algebra: Nauces, determinants, system of in gervales and eigemecrs, U decampastion Calculus: Lins, contrasty and diferencia. Mexima and mina, Mean value theorem, leg, Probability: Random vatables. Uniform, noema, exponent, olson and binarl dsbutins. Maen, median, mede and standard devation, ondional probably and Bayes theorem, equations ae Linear Algebra.. << Contents 1.1 Introduction... 1.2 Algebra of Matrices... 13° Determinants... 14 Inverse of Matrix. 15 RANK OFA Mattes 1.6 Sub-Spaces Basis and Dimension 1.7 System of Equations... 18 Eigenvalues and Eigenvectors Previous GATE and ESE Question... Calculus .. 77-228 21 Limite 22 Continuity, 23. Differentiability, 24 Mean Value Theorems nnn 25 Computing the Derivative. 2.6 Applications of Derivatives... 27 Partial Derivatives sun 28 Total Derivatives: “ a 2.9 Maxima and Minima (of Function of Two independent Variables)... 210 Theorems of Integral Calculus. 2.11 Definite Integrals. 2.12. Applications of Integra 2.13. Multiple integrals and Their Applications... 214 Vectors... Differential Equation: Bd 3.2 33 34 35 Complex Functions .. 4d 42 43 44 45 46 Previous GATE and ESE Questions. Introduction a Differential Equations of First Order un. Linear Differential Equations (OF nth Order) ‘Two Other Methods of Finding Pl... . 250 Equations Reducible to Linear Equation with Constant Coefficient, 251 Previous GATE and ESE Questions. 1-252 Introduction Complex Functions. Limit of a Complex Functior Singularit Derivative of fiz. Analytic Functions... 47 Complex integration 48 Cauchy’sTheorem .. 49° Cauchy'sintegral Formula. 4.10 Series of Complex Terms.. 4.11. Zeros and Singularities ot Poles of an Analytic Function 4.12 Residues Previous GATE and ESE Questions. Probability and Statistics .. 5.1 Probability Fundamentals... 5.2 Statistics 5.3 Probability Distributions . “ Previous GATE and ESE Questions... Numerical Method 6.1 IntFOdUCHION nen _ 6.2 Numerical Solution of System of Linear Equation: 6.3 Numerical Solutions of Nonlinear Algebraic and Transcendental Equations by Bisection, Regula-Falsi, Secant and Newton-Raphson Methods 378 64 Numerical Integration (Quadrature) by Trapezoidal and Simpson's Rules...384 65 Numerical Solution of Ordinary Differential Equations. 389 Previous GATE and ESE Questions 394 Transform Theory. 7.1 Laplace Transform 7.2 Definition... 73___Transforms of lementaty Functions 74 Properties of Laplace Transforms. 7.5 Evaluation of Integrals by Laplace Transforms... 7.6 Inverse Transforms ~ Method of Partial Fractions. 7.7 Unit Step Function. : 7.8 Second Shifting Propertyun 7.9 Unit Impulse Function. 7.10 Periodic functions 7.11 FourierTransform 7.12 Drichilet’s Conditions Previous GATE and ESE Questions. Second Order Linear Partial Differential Equation: 8.1 Classification of Second Order Linear PDES... : 8.2 Undamped One-Dimensional Wave Equation: Vibrations ofan Elastic String 437 8.3. The One-Dimensional heat Conduction Equation sin. 442 8.4 Laplace Equation for a Rectangular Region. Previous GATE and ESE Question: Linear Algebra — 1.1 Introduction Linear Algebra is a branch of mathematics concer 8 or lini 'ed with the study of vectors, with families of vectors called vector spac Spaces and with functions that input one vector and output another, Certain rules. These functions are called linear maps or linear transformations an Matrices are rectangular arrays of numbers or syn defining the operations that can be for 1ecording to are often represented by matrices bols and matrix algebra or linear algebra provides the rules on such an object Linear Algebra and matrix theory 0c py an important place in modern mathematics and has apolic eeer———“—s—————_=—— ———— Solution of @ system of linear equationsin several unknowns, which olten resultwhen linear mathomatioa mode Bing, atucled fo represent physical problems. Nonlinear models can often be approximated by linear ones Other applications can be foundin computer graphics and in numerical methods ions in this chapter, we shall dis uss matrix algebra and its use in solving linear system of algebraic equations AX = B and in solving the Eigen value problem AX = AX. 1.2. Algebra of Matrices 1.2.1. Definition of Matrix A system of mm x n numbers arranged in the form of called an matrix of order mx n. @ rectangular array having m rows and n columns is HA [@idons » be any matrix of order mx n then itis written in the form [ar ap Ay ] a 2 an | A=[8jlnen = Gn Are: Horizontal lines called rows and vertical li 8 are called columr 1.2.2. Special Types of Matrices 1. Square Matrix: An mx nmatrix far which m= "(The number of rows is equal to number of columns) is called square matrix. fs also called an mrowed square matix ie. A = [aj], » The elements a, | F=f. ie. a,,. ap... are called DIAGONAL ELEMENTS and the line along which they lie is called PRINCIPLE DIAGONAL of matrix. Elements other than @,,, 2,, etc are called off-diagonal elements ie a, Lie) ft 2 3] Erample:A=/4 8 6| isa square Mavix 98 3h .0 = 2 for GATE and ESE Prelims MADE Ensy NOTE. A Square sub-matrix of a square matrix Ais called a “principle sub-matrix" if lis diagonal 2. Diagonal Matrix: A square matrix in which all ofdiagonal elements are zero is called a diagonal a0 if ie] matrix, The diagonal elements may or may not be zero, a if inj 30 0) Example: A= |0 5 0| isa diagonal matrix 909 The above matrix can also be written as A = diag {3, 5, 9) Properties of Diagonal Matrix: diag Ix, y. 2] + diag [p, a, d= diag [e+ p,y+qz4q] diag fx, . 2] x diag [p, @, ] = diag (xp, ya. 27] (diag [x. y, zl" = diag [1/x, Uy, Hz} (diag Ex, y, 27 = diag fx, y, 2] (diag [x, y, AY" = diag [x ’, 29 Eigen values of diag [x, y, 2] = x, yand z — rantot diag try. 2) =toiag te, 7, apts ayz 3. Scalar Matrix: A scalar matrix is a diagonal matrix with all diagonal elements being equal : =0 if tej aqyak if i=] [3 oo Example:A=|0 3 0) isa scalar matrix. 003 4 Unit Matrix or Identity Matrix: A square matrix each of whose diagonal elements is 1 and each of whose non-diagonal elements are zero is called unit matrix or an identity matrix which is denoted byt. Identity matrix is always square jap=0 if fej Thus asquare matrix A= [4j]is aunitmatixif a, = 1 when i= jand a, = Owhen ‘j aati inj \ is 00 fie 1 0] is unit matrix, z 0 oy Example:/, = : l J Sor er eee maDE ERS Linear Algebra 3 Properties of Identity Matrix: (@) Tis Identity element for mutilicaion, 60 itis called multipicative identity, (b) Als IA=A or @ mer # The mx n matrix whose elements are all zero is called null matrix. Null matrixis denoted by 0. Null matix need not be square, a, =0 ¥ i,j 000 , Example: 0,=|0 0 0 a-[2 on (i) looo L Properties of Null Matri (@) A+O=0+4 So, Ois additive identity. (b} A+(A=0 Upper THangular Matrix: An upper triangular matrxis a square matix whose lower offagonal elements are zero, i¢. a, = 0 whenever i > j fis denoted by U. ja =0 if i> The diagonal and upper off diagonal elements may or may not be zero, 4 ‘ d yo iP i IAAT = poe eu ALli oa = ee a _ = (alee = [Al = at So the determinant of an orthogonal matrix always has a modulus of 1 Hermitian Matrix (49 = A) Skew-Hermitian Matrix (4° = ~A} Unitary Matrix (A® = A or A4® = 1) Hermitian Matrix: A necessary and sufficient condition for a matrix A to be Hermitian is that AeA a bic} i nd Skew-Hermitian Matrix: A necessary and sufficient condition for @ matrix to be skew-Hermitian if As A, Exampl 0 i) swt dj 9. |S Skew-Hermitian Unitary Matrix: A square matrix 4 is said to be unitary if: Daa MADE EASY LinearAlgebra 9 Multiplying both sides by A, we get an alternate definition of unitary matrix as given below. Assquare maltix A is said to be unitary if AA ip Example: 4 | a is an example of a unitary matrix 1.3 Determinants 1.3.1. Definition la Let ay, 4,9, ay, 4, be any four numbers. The symbol = |" | represents the number a, ,a,.~ lar | 2 and is called determinants of order 2, The number 4,,. 2. ds), 4a» are Called elements of the determinant and the Number a, azo ~ ay, 2,2 i8 called the value of determinant, 1.3.2. Minors, Cofactors and Adjoint | Consider the determinant |1 ag! [831 G2 Aa| Leaving the row and column passing through the elements a, then the second order determinant thus obtained is called the minor of element a, and we will be denoted by M, Example: The Minor of element a,, ee a My, 2 Simiary Minor of element ay, = [2" 23} of “yer elon Aa 1.33 Cofactors The minor M,rutiplied by (-1) */is called the cofactor of element a, We shall denote tho cofactor of an element by corresponding capital etter Example: Cofactor of a, = A= (-1)'*/M, Cofactor of element a, by cofactor of element az, We define for any matrix, the sum ofthe products ofthe elements of any row or column with corresponding colactorsis equal to the determinant of the matrix, 10 REIILEEN for GATE and ESE Prelims MADE EASY fi 20 Exampl A [: 64 20 2 12-4 -12 then, coll) [: ay 2-1 8 JA] = (1x 12) 4 (24) +O x12) (4x4) + (6x 2)4(1 x4) = (2x2) + (Ox-1)+ (2x8) =20 1.3.4 Adjoint When all the elements of a matrix ‘A’ are replaced by its co-actor, then the transpose of that matrix is known as adjoint of matrix'A’. a> 6, AdjA = [CT Properties of adjoint matrix A(Ad) A) 1. AxAgA=|Apr 1 ix tag anes 1.3.5. Determinant of order n A determinant of order mhas n-row and n-columns. It has nx nelements. ao Ae ‘A determinant of order nis @ square array of nx n quantities enclosed between vertical bars yl ay an Cofactor of A, of elements a,in Dis equal to (~1)'"/times the determinants of order (n-1) obtained from D by leaving the row and column passing through element 4, x Ay cOmlA,) a a HtAis a3 x 3mattix then |Al = } Ay cof(A,,) Y As; coflA,) = YA, cot(A,)eto. i i Therefore, determinant can be expanded using any row or column. 1.3.6. Properties of Determinants 1. The value of a determinant does not change when rows and columns are interchanged. i.e. latl= lal 2. If any row (or column) of a matrix Ais completely zero, then | Al = 0. Such a row (or column) is called a zero row (or column) Also if any two rows (or columns) of a matrix Aare identical, then| Al = 0, 3. Ifany two rows ortwo columns of a determinant are interchanged the value of determinant is muttiplied by -1 MADE ERsy Linear Algebra iL 4, {fall elements of the one row (or one column) of a determinant are multiplied by same number k the value of determinant is k times the value of given determinant. lf Abe n-towed square matrix, and k be any scalar, then IkAl = K°LAl (@)_Ina determinant the sum of the products of the elements of any row (or column) with the cofactors of corresponding elements of any row or column is equal to the determinant value (b) In determinant the sum of the products of the elements of any row (or column) with the cofactors. of some other row or column is zero, a4 BG Example: a=|@ bo a be Then 4,4, +6,B,+¢,0, =A a) + B,B,+ 0,0, = 0 ayy + b,By + 0,0, =0 Ay + Dp, + 0,0, = A &A, + b,8, +0,0, = Oetc where A,, B,, C, etc., be cofactors of the elements a,, by, in D. Of another row (or column) the value of determinant thus obtained is equal to the value of original determinant. A ie, AV. 48 then [Al = |B] andA—S"_58 then [Al = [Bl |B! = |4|*|8] and based on this we can prove the folowing: fa) 1a| =(laly Proof of a: lal = |A*A*A... ntimes| = lal>lAl« [Al .. atimes = (lAly Proofofb; [44 = || 4 Nowsince, |AA*| = [Al 4+] lA lat] = 4 1 a Using the fact that A. Adj A= |] . J, the following can be proved for A, (a lAgjal = [ale-* (bo) [aay (adj (A))| = |Alin-9? 5. 6. 7. Ifto the elements of a row (or column) of a determinant are added m: times the corresponding elements 8. = [At] = 9. 2 for GATE and ESE Prelims MRE EASY 1.4 Inverse of Matrix ‘The inverse of a matrix A, exists if A is non-singular (i.e, | A] #0) and is given by the formula at = AA) ial | Inverse of a matrix is always unique, 14.1 Adjoint of a Square Matrix Let A=[aj} be any nxn matrix. The transpose Bot the matrix B = [A.J where A, denotes the cofactor of element a, is called the adjoint of matrix A and is denoted by symbol Adj A. Ag|(A) = [ool (A? Properties of Adjoint: If Abe any n-rowed square matrix, then (Adj A) A= A (Adj A)=| Al J, where J, is the nxnidentity matrix. 1.4.2 Properties of Inverse 1 2. 3 4, 5, 6 7, AAS ATAST Aand Bare inverse of each other it AB = BA (4By" = Ba (ABCy* = C181 At If Abe an nx nnon-singular matrix, then (A7)-1 = (4-1). If Abe an nx non-singular matrix then (4-")° = (49-1 For a2 x 2 matrix there is @ shortcut formula for inverse as given below [ i ay 7 le -b] eal "Ga-bal- af 1.5 Rank of A Matrix Rank is defined for any matrix A, Ime a (Ned not be square) ‘Some important concepts: 1. Submatrix of a Matrix: Suppose A is any matrix of the type mx n. Then a matrix obtained by leaving some rows and some colurnns from Ais called sub-matrix of A. 2. Rank of a Matrix: A number ris said to be the rank of a matrix A, if it possesses the following properties: (a) There is atleast one square sub-matrix of A of order r whose determinant is not equal to zero. (©) If the matrix A contains any square sub-matrix of order (r+ 1) and above, then the determinant of such a matrix should be zero. Put together properly (a) and (b) give the definition of the rank of a matrix as the “size of the largest non-zero minor” Note: (a) The rank of a matrix is < r, if all (r+ 1)- rowed minors of the matrix vanish. (©) The rank of a matrix is > , if there is atleast one r-rowed minor of the matrix which is not equal to ze10, (©). The rank of transpose of a matrix is same as that of original matrix. ie, r(A) = (A) MADE EASY Unear Algebra 13 (9) Rank of amatrix is same as the number of linearly independent row vectors in the matrix as well as the number of linearly independent column vectors in the matrix. (©) For any matrix A, rank (A) < min(m.n) i.e., maximum rank of A,,, = min(m, n) (Rank (AB) < Rank A Rank (AB) s Rank 8 So, Rank (48) s min(Rank A, Rank 8) (@)_ Rank (A!) = Rank (A) (h) Rank of amatrix is the number of non-zero rows in its echelon form, Echelon form: \ matrix is in echelon form if only i 1, Leading non-zero element in every row is behind leading non-zero element in previous row. This means below the leading non-zero element in every row all the elements must be zero, 2. _Allthe zero rows should be below all the non-zera rows This definition gives an altemate way of calculating the rank of larger matrices (larger than 3x3) more easily. To reduce a matrix to its echelon form use gauss elimination method on the ‘matrixand convert it into an upper triangular matrix, which willbe in echelon form, Then count the number of non-zero rows in the upper triangular matrix to get the rank of the matrix. () Elementary transformations do not alter the rank of a matrix, (@ Only null matrix can have a rank of zero. All ather matrices have rank of atleast one. 1.5.1 Elementary Matrices A matrix obtained from a unit matrix by a single elementary transformation is called an elementary matrix. 1.5.2 Results 1. Elementary transformations do not change the rank of a matrix, 2. Two matrices are equivalent if one can be obtained from another by elementary row or column transformations Equivalent matrices have same rank, since elementary transformations do not change: the rank. 3. The rank of @ product of two matrices cannot exceed the rank of either matrix. i.e. (48) < r(A) and (AB) sr(B) 4. Rank of sum of two matrices cannot exceed the sum of their ranks. (A+B) < r(A) + r(B). 5. IA, Bare two n-rowed square matrices then Rank (AB) > (Rank A) + (Rank 8) ~n. 1.6 Sub-Spaces : Basis and Dimension 1.6.1 Introduction Amatrix can be thought of as an array of its rows as also an array ofits columns. Further a row as well as column is an ordered set of numbers. This view of matrixas an array of ordered sets of rows and columnsis very Useful in dealing with various linear problems, This chapter will be devoted to consideration of such ordered sets of numbers. 1.6.2 Vector Definition: An ordered muple of numbers is called an n-vector. Then numbers which are called components of the m-vector may be written in a horizontal or in a vertical ine, and thus a vector will appear either as a row ora 4 for GATE and ESE Pretims MADE EASY column matrix. A vector whose components belong toa field Fis said to be over F. A vector over the field of real ‘numbers is called a Real vector and that over the complex field is called a complex vector. Then-vector space: The set ofall n-vectors over affld F, to be denoted by V,(P)is called the r-vector space over F, The elements of the field Fill be known as scalars relatively to the vector space. 1.6.3 Linearly dependent and Linearly Independent Sets of Vectors 1.63.1 Linear dependence and independence of vector Vectors (matrices) X,, X, XX, are said to be dependent if, 1. Allthe vectors (row or column matrices) are of same order. 2, nscalarsi,, Ao, ..),(notall zero) exists such that A,X, + ApX,+ .. +A, X,= Ootherwise they are linearly independent 1.63.2 Dependence / independency of vector by matrix method 1. Ifthe rank of the matrix of the given vectorsis equal to number of vectors, then the vectors are linearly independent 2, Ifthe rank of the matrix ofthe given vectors is ess than number of vectors, than the vectors are linearly dependent 1.6.3.3 A vector as a Linear Combination of a Set of Vectors Definition: A vector & which can be expressed in the form | = kb, +. + KE,| is said to be a linear Combination of he set, Eos of vectors, Example: Given a linearly dependent set of vectors, show that at least one member of the set is a linear combination of the remaining members of the set. Example: 4. Show-that the 23 | fromatinearly independent set, = 2. Show that the set consisting only of the zera vector, O, is linearly dependent. Solution: 1. Consider the relation A(t 2.3] + [2-20] - zero This relation is equivalent to the orclinary system of liner equations k, + ky = 0, 2k, ~ 2k, = 0, 2k, = 0 Ask,= 1 = 0, k= Oare the only values of k,, k, which satisfy these three equations, we see thatthe given set is linearly independent. 2, Let X= (0,0, 0....,0) bean n-vector whose components are all zero. Then the relation kX = Qis tue for ‘some non-zero value of the number k, For example 2x = 0 and 20. Hence the vector 0 is linearly dependent. 1.6.4 Some properties of early Independent and Dependent Sets of Vectors In the following, itis understood that the vectors belong to a given vector space V,(F). 1. Iftisa linear combination of the set (&,,....6J, then the set (1, &,, E>-.6,1is linearly dependent wehave ne = T= Keb — Kak on “KE, = 0 Abs + hot. ARE, MADE EASY Linear Algebra 15 sat least one of the coefficients, viz., that ofn, in this latter relation is not zero, we establish the linear dependence of the set I, byt 2. Also, IF{E,,....6,) is @ linearly independent and {&,,....,, n] is a linearly dependent set, then m is a linear combination of the set (&,....€)) 3, Every super-set of a linearly dependent setis linearly dependent. 4, Itmay also be easily shown that every sub-set of a linearly independent set is linearly independent, 1.6.5 Subspaces of an N-vector space V, Definition: Any non-empty set S, of vectors of V,(F) is called a subspace of V,(F), if when 1. &,,,are any two members of S, then &, + is alsoa member of 8; and 2, Gisamember of S, and kis a scalar, then K&is also a member of S. Briefly, we may say thata set Sof vectors of V,(F) isa subspace of V,(F)it closed wrt the compositions of “addition” and "multiplication with scalars" Every subspace of V,, contains the zero vector; being the product of any vector with the scalar zero. Example: & = [2, 6, c] is a non-zero vector of V, Show that the set of vectors KE is a subspace of Vai k being variable. 1.65.1 Construction of Subspaces Theorem 1: The set 5, ofall linear combinations of a given set of rfixed vectors of V, is a subspace of V, Def.1 Asubspace Spanned by a Setof Vectors. A subspace which arises as a setofall linear combinations of any given set of vectors, is said to be spanned by the given set of vectors. Def. 2. Basis of a Subspace. A set of vectors is said to be a basis of a subspace, if 1. the subspace is spanned by the set, and 2. the setis linearly independent. Itis important to notice that the set of vectors = (100... 0}, = [010..0),....@,=[00...01] is a basis of the vector space V,, for, If Kye, + ep +. + k,6,=0 then, i, 0 so that the set is linearly independent and any vector B= [yA oon ed of V, is expressible as E= a0, + 4,0, +...4+.4,0, ‘Theorem 2: A basis of a subspace, S, can always be selected from a set of vectors which span S. Let Bron Bd be a set of vectors which span a subspace S. Ii this set is linearly independent, then it is already a basis. In case itis linearly dependent, then some member ofthe set is a linear combination of the preceding members, Deleting this member, we obtain another set Which also spans S. Continuing in this manner, we shall ultimately, in a finite number of steps, arrive at a basis of S. |UNOTE: It has yet to be shown that every subspace, 6, of V, possesses a basis and that the number of veotors © inevery basis of is the same. 16 for GATE and ESE Prelims mADE EASY 1.6.6 Rowand column spaces of a matrix. Row and column ranks of a Matrix Let A, be any mx n matrix over a field F. Each of the mrows of A, consisting of m elements, is an n-vector and is as such a member of V;{F). The space spanned by the mrows which is a subspace of Vis called the Row space of the mxnmattix A, ‘Again each of the n columns consisting of m elements is an m-vector and is a member of V,(F) The space spanned by the n columns which is a subspace of V, is called the Column space of the m xn matrix A, The dimensions of these row and column spaces of matrix are respectively called the Row rank and the Column rank of the matrix Theorem #: Pre-multiplication by anon-singular matrix does not alter the rank of a matrix. Ina similar manner, we may prove that post-multiplication with @ non-singular matrix does not alter the ‘column rank of a matrix. 1.6.6.1 Equality of row rank, column rank and rank Theorem 2: The row rank of a matrix is the same as its rank Theorem 3: The column rank of a matrix is the same as its rank. Corollary 1: The rank of a matrix is equal to the maximum number ofits linearly independent rows and also to the maximum number of its linearly independent columns. Thus a matrix of rank r, has a set of r linearly independent rows (columns), such that each of the other rows (columns), is a linear combination of the same Corollary2: The rows and columns of an n-rowed non-singular square matrix form linearly independent sets and are as such bases of V,, 1.6.6.2 Connection between Rank and Span Asetof nvectors X;, X %, .. X,spans Rif they are linearly independent which can be checked by constructing ——amatrixwith X,,%,%,...X,a8 its raws (or colurins) and checking thatthe rank of sucha malticisindleectn-f however the rank is less than n, say m, then the vectors span only a subspace of R", Example: Check ifthe vectors [1 2 -1]. [2 3 Oo], [-1 2 5] span Ae Solution: 12 Step 1:ConsituctamatrixA=|2 3 0 125 Step 2: Find its rank 12-4 Since 2 3 0] = 115-0)-2(10-0)- 14 +3) 125 = 15-20-7 =-12 #0 So, rank = 3 =. The vectors are linearly independent and hence span AS, Example: Cheok ifthe vectors [1 2 3], [4 5 6] and|7 8 9] span A® Solution: Since, ean mabe EASY Linear Algebra 7 123 hasa lAl=}4 5 6 789 = 1(45- 48) - 2(36- 42) + 3(82-45) =0 So its rank 2 3 since, 7 J-5-8--s90 4 5 Rank 2 Sothe vectors [1 2 3], [4 5 6] and [7 8 9] spana subspace of A® but do not span A. 1.6.7 Orthogonality of Vectors 1. Two vectors X, and X; are orthogonal if each s non zero and the dot product X/ X, = 0. Example: Theveciors (2 6 | and [4 ¢ #] are onthogonalit [a > ofxfa e =o ie ad+ be + of = 0 Example: The vectors [1.2] and [-2 1] ere orthogonal since ‘ 1 [i ofxte qe Al xf2q = (1x-2)4(2«1) 7 Example: The vectors [1 2 9] and [-1 2.5] arenot orthogonal since (1 x-1)+ (2 2)+ (3x5) = 1840 2. Thee vectors X,, X, and X, are orthogonal if each is non zero and they are pairwise orthogonal. ie anc and % = 0 Example: The vectors [1 0 O},[0 1 G] and [0 0 4] are orthogonal since [1 0 offo 4 oO} = fo 0 oj and fo 1 off0 0 4} = fo o gj and {10 off0 0 } = fo 0 o} Hn vectors X,, X. XX, €ach of which is in R", are orthogonal, then they are surely linearly independent and hence'span A” and therefore form a basis for A. Example: The vectors [1 0 0], [0 1 0] and [0 0 1] are orthogonal and hence are linearly independent and hence span F®. They form a basis for F®. The vectors [0 -2], [-2 0] are orthogonal and hence are linearly independent and span A? and form a basis of R?. 18 GEESE for GATE and ESE Prelims MADE ERsy 4. The setofn vectors (a) orthogonal and (0) ifeach vector has unit length Xo XX, a4 called orthonormal if they are t The two conditions together can be written as j , tit is; xX, a={f it ie] A set of orthogonal vectors Xcan be converted to a set of ‘orthonormal vectors by devising each vector in the orthogonal set by its length (Euclidean norm II x]l}). Example: The set [1, 2, 4], [2, 1,4] and [3, -2, 1] is an orthogonal basis of vectors for RS, since these are pairwise orthogonal and hence are linearly independent and hence span A’. To convert this set to an orthonormal basis of 2, we need to divide each vector by its length llall = Jt¥457 =e lll = JaeT+76 = ai Hull = (Oras = ya So an ott J basis of Ais (te, 2, 4) (24-4) (3 2 1) Se an ottonormal basis of P's (“es gs Je) « et ar yoa) and (Jeg Few 1.7 System of Equations 1.7.1 Homogenous Linear Equations Suppose, os BF iat F ee Aon + Apaxy + 0) Britt yoke tn. 's.a system of mhomogenous equations in n unknowns 2.9, «.%, [iy Barney Let As fate — 8 Fre Bo Nine 4 Bea |g Lindos 0 0 o MADE EASY Linear Algebra 19 where A, X, Qare mxn, nx 1, mx 1 matrices respectively. Then obviously we can write the system of equations in the form of a single matrix equation AX=0 (ii) The matrix As called coefficient matrix of the system of equation (i. The set $= x, =0, x= 0, ...x, =O} ie., X= 08 always a solution of equation (i) But in general there may be infinite number of solutions to equation (i) Again suppose X, and X, are two solutions of (i). Then their linear combination, A,X, + RX, when Ry and A, are any arbitrary numbers, is also solution of (i. 1.7.1. Important Results The number of linearly independent infinite solutions of m homogenous linear equations in n variables, AX=0,is (n=), where ris rank of matrix A. ‘n~ ris also the number of parameters in the infinite solution, 1.7.12 Some important results regarding nature of solutions of equation AX =O Suppose there are m equations in n unknowns. Then the coefficient matrix A will be of the type m xn. Let r be rank of matrix A. Obviously r cannot greater than n, Therefore we have either r= nor ren, Case 1:Inconsistency: This is not possible in a homogeneous system since such a systemis always consistent (since the trivial solution X= [0, 0, 0...) always exists for a homogeneous system). Case 2: Consistent Unique Solution: If r= X= (0, 0,0... Note: That r= n= |A| #0i.e. Ais non-singular the equation AX = O will have only the trivial unique solution Case 3: Consistent Infinite Solution: r< nwe shall have n— rlinearly independent non-trivial infinite solutions. ‘Any linear combination of these (n- r) solutions will also be a solution of AX = O. Thus in this case, the equation AX = Owill have infinite solutions, Note: That r< 9 =7|,4|= i.e. Ais a singular matrix. 1.7.2. System of Linear Non-Homogeneous Equations Bypny + Aare + oot Api = Dy Bait Beaty nee + Angin = Dy son aw Bt + Aaa teen Army = On be a system of mnon-homogenous equations in n unknown, x4.) «Xp, A Apeniy |] Itwe write Az | Ben x ado 20 for GATE and ESE Prelims mabe ERSu f Planet 6 where A, X, Bare mx.n, nx 1, and mx 1 matrices respectively. The above equations can be written in the form of a single matrix equation AX = B. "Any set of values Of x,, x2... X_ which simultaneously satisfy all these equation is called a solutions of the system. When the system of equations has one or more solutions, the equation are said to be consistent otherwise they are said to be inconsistent". A AQ By 5] By Bop Bay by The matrix [A 6} = Bri Beem On] is called augmented matrix of the given system of equations, Condition for Consistency: The system of equations AX = Bis consistent ie., possess a solution if the Coefficient matrix A and the augmented matrix {A B] are of the same rank. Le. r(A) = 1(A, B). Case 1: Inconsistency: If (A) # r(4| B) the system AX = B, has no solution. We say that such inconsistent. tem is Cases 2and 3: Consistent systems: Now, when (A) = r(A| B) = r, The system is consistent and has solution. We say, that the rank of the system is r. Now two cases arise. Case 2: Consistent Unique Solution: If r(A) = (| 8) = r= n (where nis the number of unknown variables of the system), then the system is not only consistent but also has a unique solution, Case 3: Consistent infinite solution: if r(A) = r(A|B) = r< n, then the system is consistent, but has infinite numiberof solution ~ In. summary we can say the following: 1. Hr(A) #114] B) (Inconsistent andl hence, no solution) 2. Ifr() = (Al 8) = r= n (consistent and unique solution) 3. Ifr(A) = (|B) = 22-8h~10 = By Cayley-Hamitton theorem =0 ipa = = At-3a t= Gla? -sal Pre-multipying by A“ we get aie dia-si] = Uf 24 for GATE and ESE Prelims MADE EASY 1.85.2 Finding Higher Powers of a Matrix in Terms of its Lower Powers i Example:it A= | , a) estes 4° as a linear polynornial in A Characteristic equation is -3h-10 = 0 By Cayley-Hamilton theorem, # 34-101 = 0 = A = 3A4 107 If Ais nx nmatrix, any power of A can be written as a polynomial of maximum degree n= 1 Here, since Ais 2 x 2, we can write any power of A as a polynomial of degree 1, ie., a linear polynomial of ‘A, as shown below. A = 9A 10F AP = 3A 104 Substituting (i), again in (i), we get AP = (BA+ 10) + 10A= 1944307 Now = 194 430A Again we substitute equation (i) in equation (i) to get, At = 19(A+ 10/) + 30A=87 A+ 1907 Now A = BT AS 190A ‘Again substituting equation (i) in equation (iv) we get, AS = 87 (3A+ 101) + 190A= 451 A+ 870 _ Which is the desired result, (i) (ii) iii) (iv) 1.853 Expressing Any Matrix Polynomial in A of size n x n as a Polynomial of Degree n~1 in A by using Cayley- Hamilton Theorem Example: Process to express a polynomial of a 2 x 2 Matrix as a linear polynomial in A: a4 Example: Let A= (3 2| Express 245-344 4 A? - 47 as a linear polynomial in A. ‘Step 1: First ofall write the characteristic equation of A. In this case. la-al = Ry ' “12-2 = (B-2)(2-a+1 = -BL47 Thus the characteristic equation of Ais |4—A| =0 ie, is GALT = 0 w Step 2: By Cayley Hamilton theorem, matrix A satisfies the equation (). Therefore, pulling A =2 in (i) we get A-5A+7 = 0 A = 54-71 (ii) MADE EASY Linear Algebra 25 18.6 B=P 18.6.1 1.8.7 ‘Step 3: Find the 4, A*, 4° with the help of (i). In this case A = BATA => Als 5A TAR = A = SA-74 2A8 BAN + AP AT = 2(6AS— 7A) — 3At + AR AF TAS 14A® + AP Al = TISA®— 72) — 149 4 A2— 4 2148 4A? 41 = 21(54?- 7) ~ 48,2 — ar STA? 147A— 41 = S7[5A- 70) - 1474-4] = 1384-4037 => whichis a linear polynomial in A Similar Matrices Two matrices A and B are said to be similar, if there exists a non-singular matrix P such that + AP, Properties of Similar Matrices 1. Als always similar to A Proof: Since A = J'A/ and Fis always non-singular, therefore Ais similar to A, 2. If Ais similar to & then Bis also similar to A Proof: f Ais similar to Bthen 8 = P-1AP (where Pis non-singular) Pre-multiplying above equation by Pand post-muttipying by P-', we get PBP-' = PP-1 Appt = A, ie, A= PEP So Bis also similar to A. 3. If Ais similar to 8 and Bis similar to Cthen A is similar to C. Proof: A is similar to B= B= P14P (i) Bis similar to C= C = Q18Q aii) Substituting eq. (i) and (i) we get C= O"P-4PQ Now putting PQ = D, we get C= D-1AD, which proves that Ais similar to C. 4. Combining properties 1, 2 and 3 above we can say that the similarity relation between matrices is reflexive, symmetric and transitive and hence an equivalence relation 5. Similar matrices have the same eigenvalues. Diagonalisation of a Matrix Finding the a matrix D which is a diagonal matrix and whichis similar to A is called diagonalisation ie, we wish to find a non-singular matrix M such that A= MoM where Dis a diagonal matrix Condition for a Matrix to be Diagonalisable: 1. Anecessary and sufficient condition for a matrix A,, tobe diagonalisable is that the matrix must have linearly independent eigen vectors. 2. Asutficient (but not necessary) condition for a matrix 4,, 0 be diagonalisable is that the matrix must have n linearly independent eigen values This is because if a matrix has linearly inclependent eigen values then it surely has a linearly independent eigen vectors (although the converse of this is not true), 26 EPIEPIEMEMENEY for GATE and ESE Prelims MADE EASY When Ais diagonalisable A = MDM, where the matrix Dis a diagonal matrix constructed using the eigen values of A as its diagonal elements. Also the corresponding matrix M can be obtained by | constructing a nx mmatrix whose columns are the eigen vectors of A. Pra I application of Diagonalisation: One of the uses of diagonalisation is for computing higher powers of a matrix efficiently. It As M*0Mthen A* = MID" M ‘The above property makes it easy to compute higher powers of a matrix A, since computing D? is much more easy compared with computing A”, Previous GATE and ESE Questions. ICS, GATE-2003, 2 marks] 4213 Q4_ Forthe matrix (; | the eigen values are Q1 Given Matrix(4]=|6 3 4 7], the rankofthe ie io (@) Sand-s (b) -Band-5 matrix is (©) Bands (¢) Sando @4 (0) 3 [ME, GATE-2003, 1 mark] @e2 (dt [CE, GATE-2003, 1 mark] @.5. For which vaiue of » will the matrix given below become singular? Q.2 Consider the system of simultaneous equations _ Sey 7 a arty+ 22-6 402 Bey+ze5 1260 This system has a) 4 we {@) unique solution os 2 (0) infinite numipar of solutions [ME, GATE-2004, 2 marks} (6) nosolution (@) exactly two solutions Q6 Let A, B,C, Dbe nxn matrices, each with non- IME, GATE-2003, 2 marks] Zero determinant, If ABCD = J, then 8-1 is Q.3 _ Consider the following system of linear equations " Con “ 24 41x] fa (© apc 43 ~12]) y/=/5 os : (2) does not necessarily exist WV. ICS, GATE-2004, 1 mark] Notice that the second and the third columns of the coefficient matrix are linearly dependent, For @.7 How many solutions does the following system how many values of a, does this system of of linear equations have? equations have infinitely many solutions? srt Sy=-1 5 x-y=2; x48y=3 @o (4 (2) infinitely many (b) two distinct solutions 2 (Q) infinitely many (©) unique (2) none [CS, GATE-2004, 2 marks] MADE ERSY Linear Algebra 27 4-2 Q8_ The eigen values of the matrix (3 ] (b) are ~1 and 2 (d) cannot be determined ICE, GATE-2004, 2 marks] (a) are 1 anda (C) are ands Q.9_ The sum of the eigen values of the matrix given fies belowis)1 5 1 34 | (a) 6 (b)7 (9 (9) 18 (ME, GATE-2004, 1 mark] Q.10 Consider the matrices X43). Yeas) ANd Py The arder of [P(X"Y)* PT} will be (a) (2x2) (b) (3 x 3) (c) (4x3) (d) (3x 4) [CE, GATE-2005, 1 mark] Q.41 Given an orthogonal matrix 114d ttt] peas Amle sg of 4] O00 1 -1 1 1 il [1 “re meses 1 1 o foo Jo S00 1 1 Ss oO oO Joos oO | 1 1 i 0 i oo (ee 2 aioe gy @ltooo o100 1 eee 0700 ooot 1 oo jo 1 900; [EC, GATE-2005, 2 marks) 10-1 @t26R=|2 1) then top row of Ris Baie @6 6 4) 6 31 (2 0 -) (d) [2-1 12} (EE, GATE-2005, 2 marks} 2 OF ta Q.43 Let. A F ‘ ] ana At=|2 0b Then (at b) = 7 3 fa) 2 (b) 20 19 " fo) bo ® oo [EC, GATE-2005, 2 marks] Q.14 Consider a non-homogeneous system of linear equations representing mathematically an over- determined system. Such a system will be (@) consistent having a unique solution (6) consistent having many solutions (©) inconsistent having a unique solution (2) inconsistent having no solution ICE, GATE-2008, 1 mark] Q.15 Ais a3 x 4 real matrix and Ax = bis an inconsistent system of equations, The highest possible rank of Ais @t 3 (b) 2 (d)4 (ME, GATE-2005, 1 mark] Q.16 In the matrix equation Px = g, which of the following is a necessary condition for the existence of at least one solution for the unknown vectorx (a) Augmented matrix [Pq] must have the same. rank as matrix P (b) Vector q must have only non-zero elements (¢) Matrix P must be singular (d) Matrix Pmust be square [EE, GATE-2005, 1 mark] Q.17 Consider the following system of equations in three real variables x,, x, and x, Bry nx, + Bx = 1 Sr, ~ 2x, + 5x, = 2 4x + 1x3 = 3 28 This system of equations has (@) nosolution (©) aunique solution (6) more than one buta finite number of solutions (d) an infinite number of solutions [CS, GATE-2008, 2 marks] Q.18 Which one of the following is an eigen vector of 5000 the matrix 50 > 0021 oe eee 1 0 2 oO @ |, 9 [ME, GATE-2005, 2 marks] 3-2 2 ———@:49-Forthe matricA= one of the eigen — 004 values is equal to -2. Which of he followings an eigen vector? 3 “3 fa) |-2 ()] 2 1 -1 7 2 (o) |-2 ( |5 3 0 [EE, GATE-2005, 2 marks] -4 2] 4 ay () [3 wl {EC, GATE-2005, 2 marks} ; (@ () EXCEPTS for GATE and ESE Prelims MADE EASY Q.21 What are the eigen values of the following 2 x 2 f2 1] [2 sl (@) -1 and 1 (b) tand6 (©) 2and5 (d) 4and-1 ICS, GATE-2005, 2 marks} 0.22 Consider the system of equations Ay. Spey = Moxy Where, A is a scalar. Let (A, x) be an ceigen-pair ofan eigen value and its corresponding eigen vector for real matrix A. Let / be a (7 1) unit matrix. Which one of the following statement is NOT correct? (@) For a homogeneous 1 x n system of linear equations, (A - A/} = 0 having a nontrivial solution, the rank of (A ~ Ad) is less than n (b) For matrix A", m being a positive integer, G,", xp"willbe the eigen-pair for all (©) IFAT = AM then |,] = 1 for all’ (0) IFAT = A, then A, is real for all i [CE, GATE-2005, 2 marks] Q.23 Multiplication of matrices E and Fis G. Matrices Eand Gare f ne - — Es| sino cos 0] andG@=|0 1 0 o 014 oo4 What is the matrix F? ‘cos ~sind 0) (a) | sine cosa 0 o 0 4 cos8 cos8 0) (b) |-c0se sind 0} o oo ‘| cosé sind 0 {o) |-sino caso 0 o 01 sine cose 0 (@) |cose sind 0 o 01 [ME, GATE-2006, 2 marks] | Linear Algebra 29 Q.24 Match List-| with List-II and select the correct answer using the codes given below the lists: List-1 Singular matrix Non-square matrix Real symmetric Orthogonal matrix List- 1. Determinant is not defined 2. Determinantis always one 3. Determinantis zero 4 5. yom> Eigen values are always real . Eigen values are not detined Codes: A BC D @3 1 4 2 2 3 4 4 @3 2 5 4 @3 4 2 4 IME, GATE-2006, 2 marks] 14 Q.25 The rank of the matrix |1 1 0! is L144 (@o (ot 2 a3 [EC, GATE-2006, 1 mark} 07-2 oF Q.26 P=} -1 [3 andR=|-7] are three 3 Le 12 vectors. An orthogonal set of vectors having a span that contains P. Q, Ris [6] [4 @ [| [3 [2] ‘| |: 14] [41} [3 [5] -3] 79 “HEE 4] [1] fs of El 3 a} 13} [4] EE, GATE-2006, 2 marks] Q.27 The following vector is nearly dependent upon the solution to the previous problem 8 2 fa) |9 (o) | 17 3 30 4 13 (e) |4 (d)] 2 5 3 [EE, GATE-2006, 2 marks] Q.28 Solution for the system defined by the set of equations 4y+3z= 8; 2r—z=2and 3x + 2y=5 is (@)x=O;y {O) x = 0; y= 112; Ox=ty=12 (0) non-existent z=4/3 2 2 ICE, GATE-2006, 1 mark] [4 2 Q.29For the matrix |> 7] the eigen value 401 corresponding to the eigen vector [ro is (2 we (b) 4 8 [EC, GATE-2006, 2 marks) 2-23 Q.30 Fora given matrix A=]-2 1 °. one of the 2210; eigen values is 3, The other two eigen values are fa) 2,-5 (b) 3, -5 (0) 2.5 3,5 ICE, GATE-2006, 2 marks) 12 Q.31 Eigen values of amatrix § = ;] are 5 and 1 What are the eigen vatues of the matrix S= 88? @ tand25 (6) Sand 1 (b) Ganda (d) Zand 10 [ME, GATE-2006, 2 marks] 30 EICCEMEEMEL for GATE and ESE Prelims MADE EASY Q.32 The eigen values and the corresponding eigen vectors of a2 x 2 matrix are given by Eigen value Eigen vector a4 [4] The matrix is 6 2 46 @) [ al ©) [: ‘| fo 4 4 8 Oy ‘| @ [: ; [EC, GATE-2006, 2 marks] Q.33 [A] is square matrix which is neither symmetric nor skew-symmetric and [4] sts transpose. The sum and difference of these matrices are defined as [S] = [A] + [A]! and (D} = [A] - [A]! respectively. Which of the following statements is TRUE? (a) Both [S] and [0] are symmetric (0) Both [S] and [D] are skew-symmetric () [Sis skew-symmetric and [D] is symmetric (4) [S]is symmetric and [D} is skew-symmetric ICE, GATE-2007, 1 mark] .84 The inverse of the 2 x 2 matrix [3 7 is 72 a7 2 @ iG 4] o i al ap 7-2 fr =| Q Ai 1 ) 3-5 -1 [CE, GATE-2007, 2 marks] Q.35 X= [x Then x n matrix, {a) has rank zero (0) is orthogonal x,]7 is an Mtuple nonzero vector. =x (b) has rank 1 (9) has rank n (EE, GATE-2007, 1 mark} Q.36 It is given that X,. X, .. Xy are M non-zero, orthogonal vectors. The dimension ofthe vector ‘space spanned by the 2M vectors Xj... Xp AK, Kyo Kyi (@) 2M (b) M41 mM (d) dependent an the choice of X,. Xp. Xyy [EC, GATE-2007, 2 marks} Q.37 Consider the set of (column) vectors defined by X= (re Pts, +x, + x5 = 0, where x? =[x,.x5, %4]"]. Which of the following is TRUE? (@) ((1, “1, OF, (1, 0, 17] is @ basis for the subspace X. (©) {[1,-1, 0)" [1,0,-4P) isalinearly independent set, but it does not span X and therefore is not a basis of X (c) Xis not a subspace for F® (a) None of the above ICS, GATE-2007, 2 marks} Q.38 For what values of « and B. the following simultaneous equations have an infinite number of solutions? vayt2s5 y+ 3y+3z=9 y+ 2yraz=p (a) 2.7 (b) 3.8 (0) 8.3 () 7.2 ICE, GATE-2007, 2 marks] .39 Thenumier of linearly independent eigen vectors a f is off sls (a0 2 (o)1 (A) infinite IME, GATE-2007, 2 marks} Q.40 The linear operation L(x) is defined by the cross product L(x) = b x X, where b=[0 1 0)’ and Xz [x xg¥g]"are three dimensional vectors, The 3x3 matrix i of this operation satisfies " Lol= oe %3 Then the eigen values of Mare (a) 0,44, ~1 (1-41 (hi-id ()i.-0 [EE, GATE-2007, 2 marks) Q.41 The minimum and the maximum eigen values oh 113) the matix] 1 5 1| are-2and 6, respectively 344] What is the other eigen value? (a) 5 (b)3 (4 @a [CE, GATE-2007, 1 mat mabe EASY Linear Algebra 31 Q.42 Ifa square matrix A is real and symmetric, then the eigen values (@) ate always real (©) are always real and positive (c) are always real and non-negative (@) occur in complex conjugate pairs [ME, GATE-2007, 1 mark] Statement for Linked Answer Question 43 and 44. Cayley-Hamilton Theorem states that a square matrix satisfies its own characteristic equation. Consider a matrix 3 2 Alo Q.43 A satisfies the relation (a) A+ 314 281 =0 (b) AP4 244 27-0 (©) (AFD (A+ 20) =1 (ch) exp (A)=0 {EE, GATE-2007, 2 marks} 0.44 A equals (@) 5114+ 5101 (0) 154 A+ 1557 (©) 809.4 + 1047 (a) exp (94) (EE, GATE-2007, 2 marks] Q.45 The product of matrices (PO)"P is (a) Po (a7 (©) P1Q"P (¢) PQP* ICE, GATE-2008, 1 mark] Q.46 A is mx ntull rank matrix with m> nand Tis an identity matrix. Let matrix 4’ = (ATA) AT, Then, which one of the following statement is TRUE? (a) ANA= A (b) (AANA (©) ANAS () AWA = [EE, GATE-2008, 2 marks] Q.47 It the rank of a (5 x 6) matrix Qis 4, then which one of the following statements is correct? (@) Qwillhave four linearly independent rows and four linearly independent columns (b) Quill have four linearly independent rows and five linearly independent columns (©) QQ" willbe invertible (d) O'Owillbe invertible [EE, GATE-2008, 1 mark] Q.48 The following simultaneous equations xtyez=3 xt 2y+3z04 xtaytke=6 will NOT have a unique solution for k equal to @o (5 m6 (7 ICE, GATE-2008, 2 marks] Q.49 For what value of a, if any wilthe following system of equations in x, y and z have a solution? 24 Bya4ixtyez=4ix42y-z0a (a) Any eal number (b) 0 (1 (@) Thereisno such value [ME, GATE-2008, 2 marks] Q.50 The system of linear equations 4x4 2y=7 ary =6 has {a} aunique solution {b) no solution {6) an infinite number of solutions (@) exactly two distinct solutions [EC, GATE-2008, 1 mark] Q.51 The following system of equations Hy Ay Oey = x, +21, + 8x, = 2 x) +4, + ax =4 has a unique solution, The only possible value(s) for aislare 0 (0) either 0 or + (©) one of 0,1 or -1 (@) any real number other than 5 [CS, GATE-2008, 1 mark] 4 Q.82 The Eigen values of the matrix {P] [ ei 4 are (@) -7and8 (co) Sanda (©) Gand 5 (A) tand2 [CE, GATE-2008, 2 marks] ie Q.53 The eigen vectors of the matrix [ a 3 are written inthe form (:] and is} What is a + b? a\ "|p 0 (b) 12 (4 2 IME, GATE-2008, 2 marks] 32 for GATE and ESE Prelims Q.54 How many ofthe folowing matrices have an eigen value 1? 1 O}fo aft =f 10 0 offo offs 1Jar4} + (@) one (0) two ©) three (@) four (CS, GATE-2008, 2 marks] * 124 Q.55 Thematrix|3 0 6]has one eigenvalue equal 14p 103, The sum of the other two eigen values is @oe (b) p-1 () p-2 (6) p-3 7 IME, GATE-2008, 1 mark] Q.56 All the four entries of the 2 x 2 matrix Ps [p ae ] are nonzero, and one ofits eigen Pas Poa | values is Zero. Which of the following statements. is true? (8) Py Pao ~ Pra Pay = 1 (©) Pys Pro ~ Pre Pot (©) By, Pro ~ Pig Pay = 0 (2) Py: Pya+ PrePys =0 Q.57 The characteristic equation of a (3 x 3) matrix Pis defined as ala) = |P-al] =19 432+ 224120 If J denotes identity matrix, then the inverse of matrix Pwill be (a) (P? + P+ 20) () -(P°+P +1) ()(PP+ P+ 1) (@) -(P2 + P+ 2H [EE, GATE-2008, 1 mark] Q.58 A square matrix Bis skew-symmetric if () B= 8 () 8" = Br [CE, GATE-2009, 1 mark] Q.59 For a matrix (MJ = |. the transpose of the x matrix is equal to the inverse of the matrix, [M]" = [M}". The value of x is given by 1/3 +28 EC, GATE-2008, +. mark] — a MADE ERsu ot 3 Qs (@) = (ME, GATE-2009, 1 mark] Q.60 The trace and determinant of a 2 x 2 matrix are known to be -2 and -36 respectively. It eigen values are (@) ~30 and 5 (b) -37 and~1 (©) -7and 5 (d) 17.5 and-2 [EE, GATE-2009, 1 mark} Q.61 The eigen values of the following matrix are 13 5) -3 -16 003 fa) 3,345,6-f/ (db) -64+5,34),.3-j (0) S443-f 54) (3-143; 1-9) [EC, GATE-2009, 2 marks) B+ i Q.62 The inverse of the matrix [ ae Afe-a i © 1 343 tavern 1 Ota i 3-2 1[3-2 = | OL: sez (CE, GATE-2010, 2 marks) Q.63 For the set of equations ay + Or tay + dx, = 2 Bx, + Ory + Bry + 1244 = the following statement is te: (a) Only the trivial solution », exists (6) There areno solution (©) Aunique non-trivial solution exists (d) Multiple non-trivial solutions exist [EE, GATE-2010, 2 marks) Linear Algebra 33 Q.64 One ofthe eigen vectors of the matrix A £ 3 13 Fs of o {i oo {'} IME, GATE-2010, 2 marks] 110 Q.65 An eigen vector of P=|0 2 3 is ey fo) [1 2 47 (1 1 ay (2 1 -y TEE, GATE-2010, 2 marks] | } Q.66 The eigen values ofa skew-symmetric matrix are (@) always zero (b) always pure imaginary (0) elther zero or pure imaginary (0) alwaysreal (EC, GATE-2010, 1 mark] Q.67 Consider the following matrix. A - ‘| xy. {the eigen values of A are 4 and 8, then (a) x=4y ()x=-3,y=9 Q.68 Consider the following system of equations 2x, +43 444 =0 ty-x5=0 ay 44,20 This system has (@) aunique solution (b) 10 solution (©) infinite number of solutions (d) five solutions [ME, GATE-2011, 2 marks} Q.69 The system of equations rey+z=6 x4 4y+62=20 xeayeazep has NO solution for values of 2 and yi given by @A=6y=20 b)A=6,u220 ()Ax6, () £6,220 [EC, GATE-2011, 2 mark} Q.70 Eigen values of a real symmetric matrix are always (@) positive (b) negative (©) real (d) complex. (ME, GATE-2011, 1 mark] Q.71 Consider the matrix as given below: 123 047 003, Which one of the following options provides the CORRECT values of the eigen values of the matrix? @ 14,3 ()3,7,3 ()7,3,2 1.23 ICS, GATE-2011, 2 marks] 95 Q.72 The eigen values of matrix [? a| are (a) -2.42 and 6.86 (©) 4.70and6.86 (b) 3.48 and 13.63 (d) 6.86 and 9.50, ICE, GATE-2012, 2 marks] Q73x+ 24224 2rty+ 22-5 x-yt The system of algebraic given below has (@) A.unique solution of x= 1, y= (©) only the two solutions of (x = 1, and (r= 2, y= 1, 2=0) (€) infinite number of solutions (8) no feasible solution IME, GATE-2012, 2 marks] Q.74 Let Abe the 2 x 2 matrix with elements ays = Ag ay = + Landay =—1 Then the eigen values of the matrix A'® are (@) 1024 and ~ 1024 (b) 1024y2 and ~1024/3 (©) ay and 4/3 (2) stav2 and -512V2 ICS, GATE-2012, 1 marks] 3 Q.75 Forthe matrix A= i el ONE of the normalized eigen vectors is given as 34 for GATE and ESE Prelims MADE Easy if ( 1 lo x-y #- | 5 co i (@ |o y-z a B tz 3 : 2 xy Pty? , Yo io B @\2 yer yro2? Ole 5 12 2 Yio, B, [CS, GATE-2013, 1 Mark] [ME, GATE-2012, 2 marks) Q.76 Given that a-[3 Joore[! °} the value A? is (a) 15A4 120 (6) 19A +307 (0) 17A+ 157 () 17As 20 [EC, EE, IN GATE-2012, 2 marks} Q.77 There are three matrixes P(A x2), O{2 x4) and Ala x 1), The minimum of mutiplication required to compute the matrix POR is [CE, GATE-2013, 1 Mark} Q.78 Let Abe an m xn matrix and Ban n xm matrix. It is given that determinant (1,, + AB) = determinant (J, + 84), where J, is the kxk identity matrix. Using the above property, the determinant of the matrix given below is 211 4) 1244] 11424 1142 @e2 ()5 oes (16 (EC, GATE-2013, 2 Marks] Q.79 Which one of the following does NOT equal tre ry ¥Pr 122 1ox(x+ xt] (1 yytd yet 1 2z+ 241 Tove Pe (b) 1 yet yrat 4 1zei 2a Q.80 The dimension of the null space of the matrix o1 ft 1-1 Olis -1 0 -1 fa) 0 (b)1 {c) 2 (a3 [IN, GATE-2013 : 1 mark] Q.81 Choose the CORRECT set of functions, which are linearly dependent. (a) sin x, sin? x and cas? x (b) cos x, sin.x and tan x {c) cos 2x, sin?.x and cos? x (d) cos 2x, sinx and cos x [ME, GATE-2013, 1 Mark] uation [2 21] [O]nag 7 ~iIbe] To {@) nosolution x ]_fo (b) only one solution |“ |= x} [0] (6) non-zero unique solution (@) muttiple solutions. (EE, GATE-2013, 1 Mark} Q.83 One pair of eigen vectors corresponding to the ol} ff) eff] of)f] [IN, GATE-2013 : 2 marks] Q.84 The eigen values of a symmetric matrix are all (@) complex with non-zero positive imaginary part (©) complex with non-zero negative imaginary part (c) real (a) pure imaginary IME, GATE-2013, 1 Mark] mane EASY Q.85 A matrix has eigen values 1 and -2. The 1 tT respeotively. The matrix is 14 12 ela ols 4 -1 0 oe oly 4 ) [$ 4 (EE, GATE-2013, 2 Marks] Q.86 The minimum eigen value ofthe following matrix is 31251 2] 5 127 27 5) fa) 0 (ot (o) 2 (a3 {EC, GATE-2013, 1 Mark] Q.87 Real matrices [Aly [Byes [lye [le.s:[Eleus and (FJ, are given. Matrices (B] and [é] are symmetric. Following statements are made with respect to these matrices 41, Matrix product [F}*[C]"[8] [C] [F] is a scalar 2, Matrix product [0]"[F| [D]is always symmetric. With reference to above statements, which of the following applies? {@) Statement 1 is true but 2s false (b) Statement 1 is false but 2 is true (c) Both the statements are true (0) Both the statements are false [CE, GATE-2004, 1 mark] nan a2 Q.88 Given the matrices J=|2 4 2) and 126 1 K =| 2}, the product KT Jk is _ [CE, GATE-2014 : 1 Mark] .89 With reference to the conventional Cartesian (x,y) Coordinate system, the vertices of a triangle have the following coordinates; (x,, y,) = (1, 0); (xy, Yo) = (2, 2h: (ty, Yq) = (4, 8). The area of the triangle is equal to Linear Algebra 35 3 3 @> OF 4 5 OF @F ICE, GATE-2014 : 1 Mark] Q.90 Which one o' the following equations is acorrect, identity for arbitrary 3 x 3 real matrices P. @and R? (a) P(Q+R)=PO+AP (b) (P~ QP = FP? 2FQ + (c) det (P+ Q) = det P+ det Q (A) (P+ Q) = P+ PO+ OP + Q? IME, GATE-2014 : 1 Mark] Q.91 Which one ofthe following statements is true for allreal symmetric matrices? (@) All the eigen values are real (0) All the eigen values are positive (0) All the eigen values are distinct (d) Sum of all the eigen values is zero. TEE, GATE-2014: 1 Mark] Q.92 For matrices of same dimension M, N and scalar c, which one of these properties DOES NOT ALWAYS hold? (@) (MyT=M (0) (oM)T = (MT (0) (M+ N)T= MT + NT (@) MN= NM [EC, GATE-2014: 1 Mark} Q.93 Which one of the following statements is NOT true for a square matrix A? (a) If Ais upper triangular, the eigen values of A are the diagonal elements of it (b) If Ais real symmetric, the eigen values of A are always real and positive (c) If Ais real, the eigen values of Aand AT are. always the same (d) fall the principal minors of A are positive, all the eigen values of A are also positive [EC, GATE-2014 ; 2 Marks} forzs 0.94 The determinantotmatrx| | ° 2 9) is 23.0 1|°— 3012 ICE, GATE-2014: 1 Mark] 36 for GATE and ESE Prelims Q.95 Given that the determinant of the matrix 13 0) 2 6 4) is—12, the determinant of the matrix 10 2 260 4 12 8). 20 4 (@) ~96 (o) 24 (©) 24 (6 IME, GATE-2014 : 1 Mark] Q.96 The determinant of matrix A is § and the determinant of matrix Bis 40. The determinant of matix AB is [EC, GATE-2014: 1 Mark] Q.97 The maximum value of the determinant among all 2x 2real symmetric matrices with trace 14 is, (EC, GATE-2014: 2 Marks] Q.98 It the matrix 4 is such that 2 -4)[1 9 5) 7 then the determinant of A is equal to [CS, GATE-2014: 1 Mark} oe | A 2 14 8 18 Q.99 The rank of the matrix is 14 -14 0 -10] ICE, GATE-2014 : 2 Marks} Q.100 Two matrices A and Bare given below: a-(? , g_[P +e pr+as] rs! lorsgs Pes? If the rank of matrix A is N, then the rank of matrix Bis (b) N-1 () 2N (EE, GATE-2014: 1 Mark] HS @s (cn Q.101 Given a system of equations r+ +222 b, Sr + y+3z= b, Which of the following is true regarding its solution? maDE ERsy (@) The system has @ unique solution for any given b, and b, (b) The system will have infinitely many solutions for any given b, and b, {c) Whether or nota solution exists depends on the given b, and b, (@) The system would have no solution for any values of 6, and b, (EE, GATE-2014 : 41 Mark] Q.102 The system of linear equations 21 3ifa] [ 5 3.0 1i/b/=|~4{has 12 5]le} [14 (a) aunique solution (6) infinitely many solutions (c) no solution (2) exactly two solutions [EC, GATE-2014 : 2 Marks] Q.103 Consider the following system of equations: ae 4 2y=1 av + 7Zz=1 veyez x7 2y 472 ‘The number of solutions for this system is ICS, GATE-2014 : 1 Mark) Q,104 The sum of Eigen values of matrix, [1M] is 215 650 795 where [M]=]655 150 835 485 355 550, (a) 915 (b) 1955, {c) 1640 (9) 2180 [CE, GATE-2014 : 1 Mark} Q.105 Consider a 3 x 3 real symmetric matrix S such that two ofits eigen values are a= 0. b #0 with “1 NY respective eigen vectors | x9 |, | yo |, Ifa ed 3} Lys then xy) + pV. + ayy equals faa (ob) b {c) ab {do (ME, GATE-2014 : 1 Mark] mADE EASY Linear Algebra 37 Q.106 One of the eigen vectors of matrix (G él is A -2 Onn ) 19 2 1 © { 44 (ME, GATE-2014 : 1 Mark] Q.107 A system matrix is given as follows, Ie 1-1 Al-6 -11 6 “6-11 5 The absolute value of the ratio of the maximum eigen value to the minimum eigen value is [EE, GATE-2014 : 2 Marks] Q.108 A real (4x4) matrix A satisfies the equation where Js the (4 x 4) identity matrix. The Positive eigen value of Ais TEC, GATE-2014 : 1 Mark} Q.109 The value of the dot product of the eigen vectors corresponding to any pair of different eigen values of a 4 x 4 symmetric positive definite mattix is ICS, GATE-2014 : 1 Mark] Q.110 The product of the non-zera eigen values of the matrix =eSoes Onaaso a) Oaas0 coos 001 is [CS, GATE-2014 : 2 Marks] Q.111 Which one of the following statements is TRUE ‘about every nxnmatrix with only real eigen values? (@) If the trace of the matrix is positive and the determinant of the matrix is negative, at least ‘one of its eigen values is negative. (b) If the trace of the matrix is positive, all its eigen values are positive. (6) If the determinant of the matrix is positive, allits eigen values are positive, (d) If the product of the trace and determinant of the matrix is positive, all its eigen values are positive. [CS, GATE-2014: 1 Mark] 478 315 Q.112 If any two columns of a determinant P= 962) are interchanged, which one of the following statements regarding the value of the determinant is CORRECT? (a) Absolute value remains unchanged but sign will change (0) Both absolute value and sign will change (c} Absolute value will change but sign will not change (d) Both absolute value and sign will remain unchanged (ME, GATE-2015 : 1 Mark] Q.113 Perform the following operations on the matrix ie 4 45) 7 9 105). [13 2 195] 1. Add the third row to the second row. 2, Subtract the third column from the first column, The determinant of the resultant matrix is ICS, GATE-2015 : 2 Marks] 1 tans Q.At4For A en i the determinant of stanx 4 AT Ais (a) seo? x (b) cose 4 (0 [EC, GATE-2015 : 1 Mark] fara 1 gay where the inverse of matrix P is Q.115 For given matrix P ie 2\* 4-3) afi 4-i) pal; aca] © Blarar 1] 443) =i 4437 =i i 4-31} © 95) alr 4-31 IME, GATE-2015 : 2 Marks) (c) Blo 38 GISEEEPMEEENES for GATE and ESE Prelims Q.116Let A=[a,]isiisn with n3 and a, =ij. The rank of Ais (a0 (1 () n-4 (0 ICE, GATE-2015 : 1 Mark] Q.117 For what value of p the following set of equations will have no solution? 2+ 3y=5 ax + py = 10 [CE, GATE-2015 : 1 Mark] Q.118 We have a set of 3 linear equations in 3 Unknowns. ‘X= Y’ means Xand Yare equivalent statements and'X # Y’ means Xand Yare not equivalent statements, P: There isa unique solution Q : The equations are linearly independent. RF : Alleigen values of the coofficiont matrix are nonzero, S: The determinant of the coefficient matrix is, nonzero. MADE ERsy Q.121 Let Abe an xn matrix with rank r(0.< r n, then none of these sytems has a solution, IIL. Im = n, then there exists a system which has a solution Which one of the following is CORRECT? (2) |, Mand lil are tue (b) Only it and tll are true (6) Only Mis true (d) None of them is true ICS, GATE-2016 : 4 Mark] 0.141 if the enties in each column of a square matix | ‘Madd up to 1, then an eigen value of Mis faa (o)3 @e2 (dt (CE, GATE-2016 : 1 Mark} 0.142 Consicer a3 x 3 matrix with every element being equal to 1. Its only non-zero eigen value is TEE, GATE-2016 : 1 Mark] Q.143 The condition for which the eigen values of the FLL matixA= | | are postive, is kot (o) k>=2 f@ kos k>-2 1 > dy ke-2 (©) k>0 (@) ke~y (ME, GATE-2016 : 1 Maric) Q.144 Considera 2 x 2 square matrix Ae [fo « lo oJ where x is unknown, If the eigen values of the atsix Aarelo+ jo to (@) + jo (0) ~jo (© +0 ()-o (EC, GATE-2016 : 1 Mark] Q.145 Two eigen values of a 3 x 3 real matrix P are (2+ Ji) and3, The determinant of Pis [CS, GATE-2016 : 1 Mark} Q.146 Considera linear time invariant system ¢= Ax ‘with intial conditions »(0) at ¢ = 0. Suppose a and b are eigen vectors of (2 x 2) matrix A Corresponding to distinct eigen values A, and Ay respectively. Then the response x(t) of the system due to initial condition s(0) = ais (a) ae" (b) eB (co) ea (@ easel [EE, GATE-2016 : 2 Marks] MADE ERSY Linear Algebra 41 Q.147 Let the eigen values of a2 x 2 matrix Abe 1, ~ 2with eigen vectorsx, and x, respectively. Then the eigen values and eigen vectors of the matrix A? 34 + 4I would, respectively, be 21x ay (0) ix, Hag ny — (©) 2, 0:4, x2 (G) 2, 0; x, + x5, x — 45 [EE, GATE-2016 : 2 Marks] Q.148 The number of linearly indepencient eigen vectors 210 ofmatrix A=|0 2 djis, 003 IME, GATE-2016 : 2 Marks] Q.149 The value of « for which the matrix eo) A=|9 7 13 | has zero as an eigen 6-4 9r value is___ [EC, GATE-2016 : 1 Mark] cage 2 3 4) whose A Q.150 Consider the matrix A = eigen values are 1, -1 and 3. Then Trace of (A 34?) is___. IIN, GATE-2016 : 2 Marks] Q.151 Suppose that the eigen values of matrix A are 1,2, 4, The determinant of (4"")"is, [CS, GATE-2016 : 1 Mark} Q.152 43 x 3 matrix Pis such that, P? = P. Then the eigen values of Pare @ 14-1 (6) 1, 0.5 + /0.866, 0.5 ~ 0.866 (c) 1,-0.5 + /0.866, -0.5 ~ 0.866 {d) 0, 1,-1 [EE, GATE-2016 : 1 Mark] Q.153 A sequence x{/7] is specified as alr] aay Lenlel d [pe mee The initial conditions are x{0] = 4, [1] = 1, and xfs} = 0 for n-<0. The value of x{12] is [EC, GATE-2016 : 2 Marks} Q.154 Let A be nx n real valued square symmetric mattix of rank 2 with 3S* 4? = 60. Consider red the following statements. I. One eigen value must be in [-6, 5] UI. The eigen value with the largest magnitude must be strictly greater than 5 Which of the above statements about eigen values of A isfare necessarily CORRECT? (@) Both tandil —(b) Fonly (©) Nonly (d) Neither I nor I ICS, GATE-2017 : 2 Marks} Q.155 The determinant of a 2 x 2 matrix is $0, If one eigen value of the matrix is 10, the other eigen value is IME, GATE-2017 : 1 Mark] 2 70 Q.156 Consider the matrix A= |79 go whose eigen vectors corresponding to eigen values 2, and 70 ) dg -80 rea = [yea] amt = [25] respectively. The value of x1 xis [ME, GATE-2017 : 2 Marks] Q.187 The product of eigen values of the matrix Pis 20 1) P=/4 3 3 O24 On (o2 os (2 [ME, GATE-2017 : 1 Mark] Q.158 Consider the matrix P= oe Which one of the following statements about P is INCORRECT? (2) Determinant of Pis equal to 1 (b) Pis orthogonal (C) Inverse of Pis equal to its transpose. (G) Allleigen values of Pare real numbers [ME, GATE-2017 : 2 Marks] 42 EIPEEEPMEGENTS for GATE and ESE Prelit 3s MADE ERsy 1-15 0 Q.159 The eigen values of thematrix A=|0 5 6 @ 0 0-6 5 -1 aro 1 1 @-1.5,6 1526 a ©1526 1.8.5 @)9 @\- [IN, GATE-2017 : 1 Mark] . Q.160 The figure shows a shape ABC and its mirror image A, 8, C, across the horizontal axis (X: [EE, GATE-2017 : 1 Mark] Q.163 The rank of the matrix axis). The coordinate transformation matrix that 14000 maps ABC to A,B,C, is 001-10 y . 01-100 onael 70001 “ mre 0001 -1 © a x is i) (EC, GATE-2017 : 1 Mark] | and veo! Rak : Q.182 Consider matrix A | | k lor which the equation AX = 0 has infnit many solutions is x | the nunoe ot istint cvaes t TEC, GATE-2018 : 1 Mai Q.183 Consider a non-singular 2 x 2 square matrix, AMtrace (A) = 4 andtrace (A?) = 5, the doterming ofthe matrix Ais__(upto 1 decimal plac [EE, GATE-2018 : 1 Mar 104) Q.184 Let 4=)-1 2 0] and B= APA? 4A loo 2 where fis the 3 x 3identily matrix, The determin of Bis___(upto 1 decimal place}, [EE, GATE-2018 : 2 Mar Q.185 Let N bea 3by 3 matrix with real number enti The matrix Vis such that N2 = 0. The eig values of Nare (a) 0,0,0 (b) 0,0,1 (0,44 444 [IN, GATE-2018 : 1 Ma Q.186 Consider two functions f(x) = (x ~ 2)? a glx) = 2x~ 1, where x is real. The smallest val of x for which f(x)= g(x) is (IN, GATE-2018 1 Q.187 Consider the following system of line equations: ax + 2ky =-2 ky + 6y= Here, x and yare the unknown and kis @ constant, The value of kfor which there are ini number of solutions is @3 (-3 4 (0) -6 [IN, GATE-2018 : 2 Mé ( Q.188 Consider a matrix A= uv? where v= | v. The largest eigenvalue of Ais __- ICS, GATE-2018 : 1 Me made EASY Linear Algebra 45 Q.189 Consider a matrix P whose only eigenvectors = the multiples o aretemutesat [| Consider the following statements: |. Pdoes nothave an inverse. Il. Phas a repeated eigenvalue. Ill, P cannot be diagonalized Which one of the following options is correct? (@) Only | and ill are necessarily tue (©) Only itis necessarily true (6) Only | and tare necessarily tue (6) Only il and Il are necessarily tue ICS, GATE-2018 : 2 Marks] 1 Q.190 Let the Eigen vector of the matrix [i 3] be written in the torm| "anal "| what is the a b value of (a+ b)? EEE! Linear Algebra LO 2 © 3 & 4 © 5. 10. f@) 11, &) 12, (b) 18 f@) 14, 18. (a) 19. (d) 20. (6) 21, (o) 22. 27. (b) 28. (d) 29. (c) 30. (b) 31. 36. (©) 37. (a) 38. (@) 39. (b) 40, 45. (0) 46. (a) 47. (@) 48. (a) 49. 54. (a) 85. (©) 86. () 57. (d) 58. 63. (d) 64. (@) 65. (b) 66. (c) 67. 72. (b) 73. () 74. () 75. (b) 76. 81. (c) 82. (d) 83, (ad) 84. (c) 85. 90. (d) 91. (@) 92. (4) 93. () 94. 99. (2) 100. (c) 101. (b) 102. (b) 108. 108. (1) 109. (0) 110. (6) 114. (@) 112. 147, (45) 118. (a) 119. (2) 120.) 121 126. (b) 127. (d) 128. (b) 129. () 130, 135. (b) 136. (c) 137. (4) 188, (b) 139, 144. (d) 145. (15) 146. (@) 147. (a) 148 153, (233) 184. (b) 155. (5) 156, (0) 157 162. (c) 163. (4) 164. (c) 165. (c) 166, 174.(0) 172. (@) 173. (@) 174. (a) 175. 180. (0.25) 181. (c) 182. (2) 183, (55) 184, 189. (4) 190. (b) 191. (c) 192, (b) 1 5 2 [ESE Prelims-2018] 3 4 Q.191 Eigen values of the matrix |-1 3-1] are 1-13 (b) 11,2 ()1,2,4 [EE, ESE-2018] @) 1,1, 144 Q.192 If the system 2e-y+3z=2 xt y+2z=2 Sx-yraz=b has infinitely many solutions, then the values of aand 6, respectively, are () -Band6 (b) Band 6 (0) Band -6 (8) Band-6 [EE, ESE-2018] @ 6&6 O 72 © & © 9% & (a),(b)and(d) 18. (6) 16. (a) 17. (b) (b) 23. (c) 24. (@) 25. (co) 26. (a) (@ 82. (@) 33. (@) 34 (@) 35, ib) @) 41. (b) 42. (@) 43. @ 44. @ tb) 50. (b) 51. (d) 52 (b) 53. (b) (@) 59. (@ 60. (c) 61. (d) 62. (b) @) 68. (©) 69. (b) 70. ©) 71. (@ (o) 77. (16) 78. (6) 79. (a) 80. (b) (6) 86. (@) 87. (a) 88. (23) 89. (a) (88) 95. (2) 96. (200) 97. (49) 98. (0) (1) 104. (a) 105. (@) 106. (@) 107. @) fa) 113. (0) 114. (c) 145. (@) 116. (by () 12. (@) 123. (2) 124, (17) 125. (6) (b) 131. (@) 132. (©) 133. ©) 134. @) () 140. (co) 141. (@) 142. (3) 143. (@) (2) 148. (1) 150. (-6) 151, (0.125)182. (¢) (b) 158. (d) 159. (c) 160. (d) 161. (a) (2) 167. ©) 168. (c) 169. (a) 170. (a) {c) 176. (c) 177. (d) 178, (bo) 179. (6) (1) 185. (a) 186. (1) 187. (©) 188. (3) 46 (EEE for ean ese Prtims ERIE aera” : a (c) Consider first 3 x 3 minors, since maximum possible rank is 3 MADE EASY WA) = 2 HALB) = 3 Since the rank of coefficient matrix is 2and rank of 424 argument matrix is 3, which is not equal. Henoe 63 4/<0 systems hasno solution ie. systemis inconsistent 210 a) The augmented matrix for the given system is 213 S 21 -4la 34 7) =0 43 -12/5 oka 12 -8|7 413i Performing Gauss-Elimination on the above matrix 6 4 7|=0 [21 -4la 21-4) « 201 Ja 9 -12]5|—25% Jo 1-4] 5-20 Ir 2 -al7] "lo ar -6\7-a aoe [12 -8|7| 3/2 ~6|7-al2 and 6 3 7} =0 21-4) « 241 92h 19 4 ~4/5-20. Since all3 x 3 minors are zero, now try 2x 2minass. 900 [Sat 4 | : 3| Now for infinite solution it is necessary that at least one row must be completely zero, 24 . Sa~1 2 s| 8-3=5%0 te — So rank 2 ______a-= 4S is the solution ___ (c) .. There is only one value of a for which infinite Given equation are solution exists, xt 2y+ ac) ae+ysez=6 xtye Given system can be written as han i 12 1x] [6 Where 21 2iy/=|6 4-h 1 141 afiz} 15. [ 1 wal 12 116 (4-ayr- Augmented matrixis|2 4 2/6 or, (4-4-1? = 0 14 4/5 of, (4-he 4-2-1) =0 By gauss elimination % G-NGB-2) = 0 [1 2 1\6 12 1/6 21 216)—B-%, 0 -3 o|-6 5. (a) 14 4/5 0 -1 o|-1 8 x0 mie [12118] For singularity of matrix = e 2 : =0 —. Jo 2 o|-6 out .0 lo o ofa| 3 80~12)~ (02% 12) = xed MADE EASY Linear Algebra 47 6. (b} XT 8x4 A.B, 6, Dis nxn matic Y> 4x3 Given ABCD =| XY 5 3x3 = ABCDD* C= DC OY! > 3x3 = 1-4 P+ 2x3 ate Pla 3x2 ° PUY) PT —> (2x8) (3x3) (8x 2)-9 22 - cals 2 (PTY PNT 5 2x2 Bini otc Hep oa 1 (6) For orthogonal matrix 10. 15 =] The avg mented matrixis | 1 ~1] 2 eG 3| Using gauss-elimination on above matrix we gel, 15 |-1 -1 5|- 1-12 o alt 1 als 0 a2 se 1 5|~i 0 4}4 0 olo Rank (Al 8] = 2 (number ofnon zero rows in [Al 8) Rank [A] = 2(number ofnon zerorows in (A]) Rank [Al] = Rank [4] = number of variables + Unique solution exists. Correct choice is (c). f=, (©) Characteristic equationis . 4-2-2 la-all=[ 0 4. (4-2) x(1=2)-[C2)x(-2)]=0 a2-52 = 0 = 0 Hence, 0, 5 are the eigen values. (b) ‘Sum of eigen values of given matrix = sum of diagonal element of given matrix = 145+ 1=7. (a) With the given order we can say that order of matrices are as follows AAT =I i.e, Identity matrix, (Aalyt = = 12. (b) fore Re[2 1 4 232 adi(A) _ [cofactor (Ay)” ide il 10-4 IAl=|2 4 -4 232 1(2 +3) -0(4 + 2)~ 116-2) =6-4=1 Since we need only the top row of A, we need to find only first column of cof (A) which after transpose will become first row of adj (A) col. (1,1 cof. (2, 1) = cof. (3, 1 cof. (A) = 5 -3 1 Adj(A) = [cof.(A)JT=|- - - Dividing by| A] = 1 gives 5-31 mel - Top row of R= [5 ~3 1] 48 for GATE and ESE Prelims MADE ERsu 13. (a) ee aaa ele fm, fo 2 v2|ue 5 [; ~0 1? Al -{} 4 oo -2|-1 o 3 jlo [7 lot Rank (Lal Bt 414, me | 2eci5] ai 0 o 3% J > lo1 = 2a-0.1b=0> 49, 4 3b=1ab=5 Now substitute b in equation (i), we gat 1 a So, avo- 4) 6073 _ 1420217 “eo * 60” 20 (a), (b) and (d) all possible. in an over determined system having more equations than variables, all three possibilities sill exist (a) consistent unique (b) consistent infinite and (d) in consistent with no situation, (b) Rank ([Al) = Since Rank ([A | B]) = Rank ([A]) = number of variables, The system has unique solution, (a) First solve for eigen values by solving characteristic equation |A-Ar| = 6-2 0 0 of no) ete eG 0 0 a-ak 14 0 0 8 i = (G-A)(G~A) (2-2) (1-2-3) 0 16. 17. nxn) = mine, So, Highest possible rank = Least value of 3 and 4. ie highest possible rank (based on size of A) =3 However if the rank of A = 3 then rank of [A] B] also would be 3, which means the system would become consistent. Butitis given that the system is inconsistent. So the maximum rank of A could only be 2 (a) Rank [Pq] = Rank [P] is necessary for existence of at least one solution to Px= g. {he augmented matix forte given system i 2 -1.3/1) 3 -2 52] “4 sl Using gauss-elimination method on above matrix we get, [2 -1 3] 1] 2-4 3) q |3 25 2 lo 0 v2 y2| v2 4 sa} 2 |o -92 s/alzr2 5-1) (5-2) (HP -3A~1) = 34 V8 2 fo 0 OE to => Sry = 0; -Bxy + x= 0 ; Bry 4x, =0 Solving which we get x, = 0, x, = 0, x, and x, may bo anything The eigen vector corresponding to written as maybe fig ol [hy |_| x] {0 xj lo where k,, k may be any real number. Since choice (a)is the only matrix in this form with both x, and.x, = 0, so itis the correct answer. Since, we already gota correct eigen vector, there is no need to derive the eigen vectof corresponding to & mape ERSY Linear Algebra 49 49. (d) The eigen value problemis [A~ AI]X = 0 Since matrix is triangular, the eigen values are A a the diagonal elements themselves namely / a | =0 4 = 8, -2 and 1. Corresponding to eigen value, 2.= ~2 letus find the eigen vector Putting A = -5 [A-ar]x =0 foot [i | [:}-(] 3-2-2 2]fn] fo e 0 2-2 1 |l»l-lo eee 0 0 0 1Aflxs} fo. pipgereaye) (i) 3, Since (i) and (ii) are the same equation we take Putting 4 = -2 in above equation we get, ooo 5 -2 2ifm) fo y= 2 0 0 Aix} =lo wy ixy 251 0 0 3x] lo ‘ Which gives the equations, = ae Br, ~2xy + 2x5 = 0 @ 0 (i) Now from the answers given, we look for any 3x, =0 (i) fz Since eq, (i) and (il) are same we have vector in this ratio and we find choice (c) | 1 is 0 @ a5 =0 i) 42 k, in eq. () we get inthis ratio 2 ° = x, = 215 k So choice (2)is an eigen vector corresponding to +. Eigen vectors are of the form Aa. n) [2I6k Since we already got an answer, there is noneed tofind the secand eigen vector corresponding to le aad. x 0 Le. xy iayay =2/5K:k:0=25:1:022:5:0 21 (0) x] [2 * 2) [3 ee The characteristic equation of this matrixis given bs] 0. b y 20. (c) la-all =0 ao) 2% 1} First, find the eigen values of A | oe |A-all =0 (2-4) (6~4)-4=0 Le 22-Th46=0 ~ [2-0 ; 4 3-4 :. The eigen values of Aare 1 and 6. ce a9 oe) ie 4) Although 2,” will be the corresponding eigen 2 A values of A”, x/" need not be corresponding eigen Corresponding to 2, = -8 we need to find eigen vector vectors, 50 for GATE and ESE Prelims MADE ERsy 23. (c) 6 Method 1: tho | Take choice (a) |~3| and cos -sind 0 s| |s E=|sine cose 0 _ Firstly these are orthogonal, as can be seen by taking their dot product 100 =-6x44-3x-246x3=0 ad G@=|0 10 ‘The space spanned by these two vectors is seomingonbon Ale cording to problem nee h-afo-2 7 cose sino 0 100 or |-siné cosé OxF =|0 1 0 4 os coe 0 4 Th 1 |-3] and | -2] contains P.O) Hence we see that product of (Ex F) is unit matrix Fr —finging-o 24. 25. 28. ‘so Fhas to be the inverse of E. Adie) ie cos sing 0 = |-sind cosé 0 oO a Feels Method 2: ‘An easier method for finding Fis by multiplying Ewith each of the choices (a), (b), (c) and (é) and hone-gi proc identity matrix G. Again the answer is (c). (a) A Singular matrix — Determinant is 2er0 8. Non-square matrix» Determinant isnot defined C. Real symmetric — Eigen values are always real DD, Orthogonal matrix Determinants always one (©) Perform, Gauss elimination ig va eae Oo 1 1-10] Bs 0 2 000 It isin row Echelon form So its rank is the number of non-zero rows in this form, ie., rank = 2 (a) We are looking for orthogonal vectors having a span that contain P, Qand 27. 28. We can show this by successively settin equation (i) to P, @and Rone by one and solving for k, and k, uniquely Notice also that choices (b), (c) and (4) are wrong since none of them are orthogonal as can be se by taking pairwise dot products, () _The vector [-2 -17 30] is linearly depen upon the solution obtained in previous questi namely [-6 -3 6] and'[4 -2 3] This can be easily cheeked by finding determir [-6 -3 6 ofl4 2 3 [2 -17 2, 6 -3 6 2 3 -17 30) (80 + 51) + 3{120 + 6) + 6-68 -4) =0 Hence, itis linearly dependent. (0) The augmented matrix for given system is 20-1 cow unsiniow |g 4 3 320 2] Ei fo4ss 204 320 8] 2 Linear Algebra 29, 30, 31. 32, 33. then by Gauss elimination procedure 20-12) 5, [20 112 04 3/8225 lo 4 3 |e 3.2 a5 0 2 3/2\2 [2.0 -1/8 a* lo 4 38 00 oj-2 For last row we see 0 = -2 which is inconsistent. Also notice that (A) = 2, while (A | B) = 3, (A) r(A | 8) means inconsistent) + Solution is non-existent for above system. (c) 42 a-ak 2 -[ dhuean=[ al a tor [101 igen vector ven eigenvector | 1, IM-Anx=0 4-2 2 7 f101) * 1 2 a-aj [toi] = = (4-2) (101) +2x 101 = 0 = A=6 (b) Th, = Trace (A) Dy tay thy = Trace (A) =2+(-1) +0 Now "423 Bthgthyet = 2 Only choice (b) satisfies this condition, (a) Why py Ag «hy are the eigen values of A. Then the eigen values of Am are A a Here, Smatrix has eigen values 1 and 5. So, S® matrix has eigen vatues 12 and 5? ie. Tand 26. (a) By property of eigen values, sum of diagonal elements should be equal to sum of values of A So, EA,shy +h, = 844 12 Trace (A) Only in choice (a), Trace (A) = 12 (d) Since s= (Asan? = ATS (ANT uu 51 =AT4A=S le. s=8 + Sis symmetric Since Dl = (A-All= AT (Ayr AT-A={A-Al)=-D ie, Dr=-0 So Dis Skew-Symmetri. (a) ab lnverse of | 5g (b) it Xe Oy. a) Rank X= 4, since itis non-zero n-tuple. Rank X7 = Rank X= 1 Now Rank (X") < min(Rank X, Rank x7) Rank (XX7) smin(4, 1) Rank (XX7)< 1, So XXThas a rank of a either 0 or 1 But since both Xand X"are non-zero vectors, so neither of their ranks can be zero. So XXThas a rank 1 (c) Since (X), Xp. Xy) are orthogonal, they span a vector space of dimension M. Sines (-X,, -X,, .. Xy)are linearly dependent on Kya Xpy oo Ky the 80 (%, Ny, Xp oo Kyp Xp Xp ~X,) will also span a vector space of dimension Monly. (a) To be basis for subspace X, two conditions are tobe satistied 1. The vectors have to be linearly independent. 2, They must span x Hee, X= (re MPlx, +x, +x,= 0) a7 be aganl” 52 for GATE and ESE Prelims MADE EASY Step 1: Now, {[1,-1,0],[1,0,-1]}isalinearly 39, (b) independent set because one cannot obtained 21 ‘rom another by scalar multiplication, The fact that a F il it is independent can also be established by [e-al=0 1-10 2-k 4 seeing that rank of [ a ‘l is2 [ : a =0 Step 2: Next, we need to check if the set spans X. = (2-2? =0 Here, X= fre AP |x, +x, 4x5 = 0) = nae Now, consider the eigen value probiem why he (A-UIX=0 ' The general infinite solution of X=| ky it I ky 0 2-a}[e| = |o} a] fo ke] [hk Put A = 2, we get, Choosing k,. Kas} "| =|] and || =|), 0 1] [x] _ fo fo] |k 6} [0 _ =|o we get 2 linearly independent solutions, for X, 2 . co Pe 0) X=} 0 jor] k 0.x, = anything le oll 4] _ [ft] Now since both of these can be generated by linear »|*[o|-*\o | combinations of (1, -1, OJ" and (1, 0, 1)", the ~ set spans X. Since we have shown thatthe setis 40. (d) rot only linearly independent but also spans X, The cross product of b= [0 1 of” therefore by definition it is a basis for the subsp. — 2 and X= [x x x3) canbewrittenas 38. (a) DAA The augmented matrix for this system is fod fi oxx=l0 4 0 11118 I te ay 13.319 : 12 a/p = it O0j-xk Using Gauss-elimination method we get = fy 0 -x] et as 11 445 x 13 3/9/-ER4Jo 2 2\4 Now Lx) = bx X= Ml xy 12 al, 0 1 a-1/p-5 xy] 11.175 where Mis a3 x 3 matrix —BsJo2 2] 4 0 0 a-2|6-7, 1 & &% Let Mz|@q o& &% Now, for infinite solution last row must be or % completely zero iea-2 and B-7=0 a = = 2andB=7 Now M)xy| = xX x mabe ERSY Linear Algebra 53 for & ca] [mi] [2s 44. (a) alec cal |x|=| 0 Tocalculate A? & ¢ clls| [Hx start from A? + 3A + 2F = 0 which has been By matching LHS and AHS we get ee 00 MTm) [xm = A 2-34-21 00 ollel=|o At = APx AP =(-BA-21)(-3A-21) 10 0llsl [-y Aa AZAS AT (-3A-21) +124 at ono 15 A-14r $0, M=|0-00 eae -10 oJ (15 A- 141) (15 A~141) Now we have to find the eigen values of M 25 AP + 420 A+ 1867 IM—nal = 0 = 225 (-8A-21)+ 420A 1967 Pree alo 5 ofeo “1 0 =A) A(-255 A~ 2541) = -AU2-0) + 40-2) =0 255 A?—254 A > M+h=0 ~255(-3 A~ 21) 2544 pee a io = 51145101 = d= O,k=4i So, the eigen values of Mare i, ~i and 0 45. (b) (Pay P= (QPP “oe Trace (A) SOME =A 1454127 cl Now, 46. (a) Choice (a) AA’A = Ais correct Since, AA’A = AIATA ATA 42. (a) = AL(ATAY ATA] The eigen values of any symmetric matrix is Let, AA=P always real Then, = AIPA P]=AsT=A 43. (a) 47. (a) If rank of (5 x 6) matrix is 4, then surety it must have exactly 4 linearly independent rows as well jana as 4 linearly independent columns, since boa rank = row rank = column rank. | 48. (d) (3-eay4 The augmented matrix for given system is 22 4 Bh 11143 ‘A.will satisfy this equation according to Caylay- 4234 Hamilton theorem ale le. AP+3A421=0 ‘multiplying by 4" on both sides we get AN AR 4+ 3AT AG 2A =0 A+ 31424 Using Gauss elimination we reduce this to an Upper triangular matrix to investigate is rank. 34 EMCEE for GATE and ESE Prelims MADE ERsy Now, Rank [A] B} = 2 14143 11°43 (The number of non-zero rows in [A 6) 12 3/4)—Bao 1 2 11 Rank (A) = 1 , 14 kI6 ete (The number of non-zer0 rows in (Al) 110113 Sinoe, Rank [Al 6] * Rank[A], —qar)o 1 2 | The system has no solution, 51. (a) Nowit k#7 ‘The augmented matrix for above system is rank (A) ark (ALB) = ae eee «unique solution Bem io k= T.tank (A) = rank (AB) =2 taper ( es which is less than number of variables 1 C when k = 7, unigue solution is not possible ee and only infinite solution is possible. jo 1 1 fi a 0 0 a-sjo} Nowastong as a—5 0, rank (4) = rank (A| B) =3 230 | ». A can take any real value except 5. Closest Poop 14/4 correct answeris () [v2 -lal 52. (b) Performing guess-elimination on this matrix, we Az ie a | ace 2-5 ee 1 [2 3 0, 4 Characteristic equation of Ais 444 |e, 0-1/2 ‘2 lesen 4 —_}1+-2~1}a} 938 o- =tte=-2}— {=o 2 3 ola = (4-2) (-5-2)-2x5=0 eon ee ee ae = 2+2-90=0 0 0 ola ae tia#0, (A)=2end (418) =3,hence system §3. (b) will have no solutions (ia) 2 | lta=0, (A) = (AIB) =2, then the system willbe | e-r)| consistent and wil have solution (Infinite solution). = (1-2) 50. (b) =1,2 The system can be written in matrix from as Now since the eigen value probiemis 4 2Ife J (A-a)X= 0 [ IG]. 6. (‘2 2 k fi The Augmented matrix [A | 8] is given by o 2-a}** lo bile] Performing Gauss elimination on this {AIB] as follows: 7 5/2. [ele] se pene (42 Ain [0 0 putting the value of A = 1 and X= X, = (:] - fill => a=0 a ae putting the value of & = 2and X [5] made EASY Linear Algebra 55 1 aift 1404 p=34hy thy ABA ee 5 en, > Ajthy=pet-3=p-2 and 0=0 56. (c) pal Since, na, = Al > “3 i) and If one of the eigen values is zero, then From equations (i) and (ii) 1a,= Al =0 11 arb= 044 ne Lal = [Pr Pe a ‘ Toe, pa 54. (a) Pry Poa Pp Poy = 0 10 Which is choice c) emsansa 4 _ I characteristic equation is 2422424120 Then by cayley -hamitton theorem, Pi + Pes 2P4J=0 I= -P8— Pp? 2p Muttiolying by P-' on both sides, Pla -pe- poy =4P2+ P+ 21) i-a 0 0 o-a) (1-ayxea 58. (a) Asquare matrix Bis defined as skew-symmetric if and only if BY = -B, by definition. 1-1 communal =| ae oF Given, MT = A li wij-e eee (1-22 41 =0 (-ap et 1a tori he tmtortsi -1-2 Elgen values of matrix are (1-0) C1-a) =0 (142% =0 heal So, only one matrix has an eigen value of 1 which loo] Correct choice is (a). 55. (c) Sum of the eigen values of matrix is = trace of matrix = sum of diagonal values present in the = matrix 56 EES MEIEGENS for GATE and ESE Prelims MADE EASY 60. 61. 62. 63, 64. (c) 2A, = Trace (A) = Ath=-2 ti) ma, = [Al = = Aa, =-35 (ii) Solving (i) and (i) we get, and A, = 5, -7. (a) Sum of eigen values = Tr()=-1+-1+3=1 $0, Bhat Only choice (d) (3, -1 + 3} (6) ‘a by! 1 ee i ‘] (ad—bej|-c a 34% 01 7" 1 3-2 iz ool © [era mal ‘ asa Af3-2 i iL i 342i (d) 14 + Oty ty + dey = 2 Sr, + Br, + Bry + 12, 3) gives 23, 363 12/6 Performing gauss-elimination on this we get 121 4|2] A-aR [1 2 1 4]2 a6 3 12/6 00 0 olo rank (A) = rank (A| 8) = So, system is consistent Since, system's rank = 1 ig les than the number af variables, only infinite (multiple) non-trivial solution exists. nae -l13 (a) Characteristic equation of Ais teeta ‘The augmented matrix is 2h 2 1 g-ap=? (2-4 (3-2)-2 = 0 Sh+d = 0 neta The eigen value problem is |A-M|x =0 [> SIE) E Putting ay = weg igen Putting A= 4 1 2][4]_ [0 1 alle} Lo. xy #220 0 x, +2r=0 ‘i Solution is x, = kx, =-2k «(4 k ie ayimys-2:4 2 s. Choice (a) is an eigen vector. (b) 11°07 Given, p=|0 2 2 003 Pis triangular So eigen values are the diagonal elements themselves, Eigen values are thereto, dys Wy =22y=3. Now, the eigen value problem is [4—Al]& =0 1-2 1 0 ][m] fo] eo alec 0 0 8-allx} lo. corresponding to this eigen value, 0 1 Ofn] fo 0 1 alle} =o 00 2x} fo Which gives the equations m=O yt 2m 20 i st2e0 i The solution is y= 0, = 0,5, = e : Hf 80, one eigen vectors # = |0] ie..x 27:4 0 : =k:0:0 i Since, none ofthe eigen vectors given in choi = matches with tis, ratio we need to proceed fu and find the other eigen veotors correspond to the other Eigen values. Now, corresponding to A, = 2, we get substituting 2, = 2, in the eigen value probl the following setof equations, MADE ERSY Linear Algebra 57 68. (c) “11 Olfx) fo The Augmented matrix ° ° : Xe} = ; 21 110 Xe. [Algj=|0 1-110 Which gives the equations, 11 0]0 66, 67, +x,=0 2x = 0 20 Solution is x5 = 0, x, | eOXy=]k] e.xy caging 1:10 | LO} Since none of the eigen vectors given in the choicesis of this ratio, we need to proceed further and find 3" eigen vector also. By putting 2 = 3 in the eigen valve problem, we get -2 1 Oifa] fo 0-412 [x =|o 0 0 Oflx fo. 2k, + x way + xy = 0 putting x, = k, we get, x, = 2k and [k % = | 2k lf i, xyixgixye 12:4 ft ‘Only the eigen vector given in choice (b) vis 1 in this ratio, So, the correct answer is choice (b). (c) Eigen values of a skew symmetric matrix are either zero or pure imaginary. (d) Sum of eigen values = Trace (4) = 2 + y Product of eigen values = [Al = 2y—3x 4+8=24y (i) 4x8 (il) a+y (i) 2y- 3x = 32 (i) Solving (i) and (ii) we getx = -4 and y= 10. 69. 70. 71. 72. Performing gauss elimination on [41 B] we get 21 140 0 1-1/0) ata, 11 0/0 21 o4 1 °3 Rank (A) = Rank (4B) = 2 <3 So infinite number of solutions are obtained (b) The augmented matrix for the system of equations is 11116 14 6120 14ain 11116 14 61 20 0.0 4-6) -20, A= 6 and p20 then Rank (A 8) = Sand Rank (A) = 2 ‘+ Rank (Al B) # Rank (A) <. Given system of equations has no solution for A= 6andy220 (c) Eigen values of symmetric matrix are always eal (a) Since the given matricis upper triangular, is eigen values are the diagonal elements themselves, which are 1, 4 and 3 [Al B}= IAB} = [R, > ,-R] (b) werent donvanactan[2 5 5 The characteristic equation is 9A 8G 5 8-a|~ (9-2) (8-2)-25 = 0 58 for GATE and ESE Prelims maDE ERSy = 217k 447 =O ‘Characteristic equation is So eigen values are, [5-2 3 = 3.48, 13.63 | + a-a}=° 73. (c) 6-AG-2)- 0 The given system is 8A +12 =0 xe 2y4z vl keh8 aeay 422 ow, to find elgen vectors: x-y+z=1 [A-vk = 0 Use Gauss elimination method as follows: S-d 3 [fm] _ fo ‘Augmented matrix is Which af ie ee le] ah el erila Put = 2 in above equation and we get (l=) 2 12 |5| gm, re r eR [3 3]fx,] _ fo teria by afb} = fo uO 124 Which gives us the equation, 0 3 ofs| B&4]0 - of-s ee O -3 0-3 0 0 0} 0 | a" and xy tm, =0 Rank (A) = Which is only one equation, Rank (A |B) x, +x) =0 So Rank (A) = Rank (A |B] = 2 Whose solution is System is consistent 4] [-# ae (*]-F] Number of variables n= 3 . ren Sone eigen vector is ir | So we have infinite number of solutions. 7 ay ——Whictrafter normatization is ==5 a a Cole 1 iE = 2 Key LK Eigen (A) are the roots of the characteristic Ven Ht) polynomial given below: ea oui | | 2 a 1a (I-A) 1-2)-1 NB. =(1=2) (14 A)-4 ‘The other eigen vector is obtained by putting the 4-2=0 other eigen value fae i “ in a value a Koonvauestaawse at VB raecney — [°? 2,1] 9] So eigen values of A® = (Y)"° and (-V2)"° Which gives, = 20 and 20 fafa] _ yo oe (i 2 -£) = 512V2 and -512\2 Which gives the equation oxy + 3x, = 0 ce ey and x, - 3x, = 0 af | Which is only one equation mt, + Sty = 0 Linear Algebra 59 76. 7 Whose solution is +H Which after normalization is lar al’ . [3] Lue [3 B i isthe only correct choice, @. since itis a constant mutiple of one the normalized vectors which is, (b) a Characteristic equation of Ais |-5-a 3 20-2 (S-A)CA)+6=0 => +5A46=0 So, A*+5A+6r=0 (by Cayley Hamilton theorem) = A = -5A- 61 Multiplying by A on both sides, we have, A= A264 = f= -5(-5A-6N)-6A = 194+30/ Sol. The minimum number of multiplications required to muttiply Arsen With B,,, i8 rnp. To compute POR if we multiply PQ fitst and then R the number of muttiplications required would be 4 x 2 x 4 to get PQ and then 4x4 x1 multiplications to mulitply PQ with A. So total multiplications required in this method is AX 2x d+AKAKT = 32+ 16 = 48 To compute POR if we multiply QR first and then P the number of muttiplications required would be 2 x 4 x 1 to get QR and then 4x2x1 muttiplications to mulitply P with QR. 78. 79. So total multiplications required in this method is 2x4x1+4x2x1=8+8=16 Therfore, the minimum of multiplication required tocompute the matrix PQRIs = 16 (b) Take the determinant of given matrix | A] = 24-9 A2— 94 1-2] 1f4- 9-124 1] HN2- 9-22-94 41- 1] -tt1-2) 21-294 11} B11] 19-1=1] + 11-240]- 1-1 +240) (4-1 (1) 4 1-1) -1 (4) =8- 1-14 =5 (a) The given matrix can be transformed into the matrix given in options (b) (c) and (d) by lomentry operations ofthe type of R KR, or C, KC, only as shown below: Option (b) jtx 2 txtt ey Ty VBE yet ed 2a Tze 244 Option (c): tx ¥% lO x-y P-¥? ty P| BR 0 yz yo? 1z2 i 2 Option (a): fix 2 2xrty Pay Ty ALB 2 yee ae 122 tz 2 Option (a): We can show the given matrix can ‘ot be converted into option (a) without doing a column exchange which will change the sign of the determinant as can be seen below: jix x ft xt xed Ty PRB yet yyeo) 42 2 1 z41 22+) 1 ox(eet) tt elt yy+) ye 1 azey 244] 60 for GATE and ESE Prelims MADE EASY 80. (b) Thus, the two eigen value of the given matrix are 1 140 LGC) Az| 1-10 Li aia) ol Ot allesa 84, (c) order of matrix = 3 (i) The Eigen values of symmetric matrix [47 = A] Rank = 2 are purely eal «: dimension of null space of A = 1 (i) The Eigen value of skew-symmetric: matrix 81. (c) Since, cos 2x = cos®x~ sin’x, therefore cos 2xis 85. linear combination of sin? and cos? xand hence these are linearly dependent 82. (d) 2 -2]f[x]_[o 1 -1]Le]} lo. 2x, 2x, = 0 ak = yak ie. x, and x, are are having infinite number of solutions. Caetae = Multiple solutions are these. from equetont) ae Ue eee From equation (i) and (iv), c= ~2and a 83, (ad) a b)_fo 1] Figen values are Aale al|-2 -3] and x, ~ it) =0 “Vand|"), sats Joe] satisfy nd x, + ity = 0 Eos clearly, (3-H) |(12-5)(6-4)-49]- 555 -A)- 14] 235-2412 (3-2) 217+ 11)-5 (11-82) + 2111 +22) B2-51h4 33 87. (a) 2 7 5A (0-1 [eo=17+42 40] -525— 5) ~14)4.205-24+ 24)=0 + 172 112-55 + 252 + 22-4 42 =0 92+ 2022-332 = 0 98-2012 + 332 G2 —20h + 33) = 2 0,182,182 So minimum eigen value is 0. Statement 1 is true as shown below. [FlThas a size 1x5 [cl hasa size S x 3 [B)hasa size 3 x 3 mabe ERSY Linear Algebra 61 88. 89. [Clhasasize3 x5 [F]hasa size 5 x 1 So[F]"[CI"[8] [C] [F] has asize 1 x 1, Therefore itis a scalar. So, Statement 1 is true. Consider Statement 2: D” FDis always symmetric. Now D’ FD does not exist since D’,,.5, Fs, and D,,.. are Not compatible for muttiplication since, Days Fox = Kins 800 Xx 4 055 does not exist So, Statement 2 is false. Sol. 3214 J=|242 126 a 2 -1 j 24 3] KT UK= [12-2 4 2]/ 2 Lt 2 6]L-a} 1 = (68-1) 2)=6+1641-23 “1 (a) Area of the triangle = Ble -y0)+ e040 ~ Yall 1 = 3|¥2-3)+28-0)+.40-2)=2)-146-) Bi | (@) (P+ Q)? = P2+ PQ+ OP + 2 = P-P+P-0+0-P+0-Q = P2+PQ+ OP + @ 92, 94, 95. (d) Matrix multiplication is not commutative. Sol, ee) 00 4 10 Interchanging column 1 and column 2nd taking transpose 0120 23-6 4 30 -8 10 12 0 =-Ix]3 -6 4 0 -8 10} eee 1x {1(-60+32)+2(0-30)] (-28 - 60) = 88 (a) Let D = ~ 12 for the given matrix 260 130 A=|4 12 8 264 204 102 (Taking 2.common fram each raw) Det(A) = (2)°xD=8 x 12 62 EXIECMEIET for GATE and ESE Prelims MADE ERsy 96. Sol. 6 0 Determinant of A = 5 Determinant of matix | 9 44{ Is not zero Determinant of B = 40 Rank is 2 Determinant of AB = [4] |B] = 5x40=200 100. (c) 7 97. Sol. al? Al rs ax 2 + +98 Let, al? i aa[P th +98) ay regs P+s? | = [A] = ab-x? There are three cases for the rank of A 7 aaa 4 a Saar a Ml rank (A) =0 ed Seve pean maxima valve 0 = Ais null. $0 B= AAT also has to be rl a ee rece eens hence rank (6) is also equal to 0. Theretore i Sonow, JAl = a(14~a) = 142-22 this case rank (A) = rank (B) Now maximizing this with respect to a, Case Il AA s4-za=o rank (4) =1 da —> A cannot be null. So B also cannot be na = 7 since B= AA and [8 = JAat| = [Al |a"l= | al? Since Al 2<0 ‘So rank (B) #0. Now since rank (A) #2 in this oa | case, | A] =0,which means that |8| = || : So rank (B) is also #2. Now since rank (8) # ‘Ata=7,wehave amaximum, The maximum value and # 2, therefore rank (B) must be equal to TET —rerafore ins case also rank (AY = rank 98. Sol Case tll: o rank (A) = 2 -a|fi 9 3} So A has to be non-singular. ie. |Al # . Therefore, [8] =|Al*is also +0. rank (B) = 2. 2 18 10 Therefore in this case also rank (A) = rank (3). Az|-4 36 -20]=>|A)=0 Therefore, inal three cases rank (A) = rank (B) 7 63 36 rank of Ais N, then the rank of matrix Bis aso 99. Sol. 101. (b) 6044 The augmented matrix for this system 2 14 8 18 [12 216] mom [1 2 2] 4 14-14 0 -10 [5 1 3b} }o -9 -7)6)-5a Fy Ry 2A + Fp Now Gauss elimination is completed, We ca £ ° i 4 that the Rank(a) = 2 14-216)4(-2) ~14- 20) (14) 0-214}4+8 10-214) +18) Rank{A | 6] is also = 2 (does not depend on¥ of b, and b,) 5 7 . 7 Rank(/A) = Rank{Al B] < Number of variables pe ene ‘Therefore the system is consistent and asin 0000 many solutions, Linear Algebra 63 403. 104, 105, 125 4 Foo Feil 5 - = Fy» Pe~2R| 9 3 anes + “i * Rank of (4| 8) = en of A < order of matrix = Infinite number of solutions are possible, Sol Given: xn 2y4T7. mx 2y +7: = = > = r+ 7z (Putx =2) 8+7z ze :. The number of solutions for this system is one, x= 2, y=-5/2 and z = ~1 is the only solution. (a) ‘Sum of eigen values = trace of matrix = 215 + 150 + 550 = 915, (a) 3.x 3 real symmetric matrix such that two of its eigen value are a0 b# 0 with respective eigen | fy vectors | x2} | Yo x3} L¥s if ae Dthen 106. 107. XWq + XgVo + XY; = 0 because they are orthogonal xy=0 (since a+ 6) vy Ia x2 a3] } yo | = 0 ~ vs (d) ‘The characteristic equation IA~All=0 “S-A 2 fe} 9 g-a}=9 or (A-6) (A +5) +11 or 4?-6A + 52-3041) oO -h-12=0 te Vie _ 187 2 — 0 or eo) Corresponding to A= 4, we have [S20] eet which gives only one independent equation, -O + 2y=0 pe 57g gives eigen vector (2, 9) [3 sb] which gives ~ x + y= 0 (only ane independent equation) Corresponding to’ a : T=] which gives (1.1) 2 So, the eigen vectors are { HI and tt Sol Characteristic equation is | A~ar| = | 6 1-k 6 6-11 5a] 64 for GATE and ESE Prelims => -ASS + 11.544 22 + 66] —1[-30 + 6A +36] 1686-66-26] = 0 “AAP +64 + 11)-1(62 + 6) + 62 = 0 W~6M- 112-6 = 0 M+ GM + 1104+6=0 (A+ 1)(A+2)(4 +3) =0 Az -1,-2,-3 Maximum eigen value is~1 of Aare || = 1,2,3. Ratio of maximum and minimum eigen value is Uudaag =3 108. Sol, Since, A? = 1, eig(A?) = eigl) = 1 = cig)? = 1 > eig(A) = 21 Therefore, the positive eigen value of Ais +1 109. Sol. The value of the dol product of the eigenvectors corresponding to any pair of diferent eigen values of a4 x 4 symmetric positive definite matrix is 0 110. Sol, = = = tpt tt ay = key = 3b = kb = k=d (i) k=0 = Eigen value k = 0 ©. There are 3 distinct eigen values: 0,2,3 Product of non-zero eigen values: 2 x 3 = 6 i 112. (a) Property of determinant : If any two rows or columns are interchanged, then mangnitude o determinant remains same but sign changes, 113. Sol Since operations: 1 and 2 are elementayy operations of the type of A, KR; and C+ KC respectively, the determinant will be unchanged from the original determinant 34 45] So the required determinant = | 7 9. 105) 13 2 195] 34 45 ao (3 4 9 7 9 105} 2218217 9 of =0 13 2 195 132 0 So the requirect determinant =0, 144, (c) Long Method: MADE Ensy (a) either the trace or determinant is positive, there exist at least one positive eigen value. Trace of the matrix is postive and the determinany of the matrixis negative, this is possible only when there is odd number of negative eigen values Hence at least one eigen value is negative, fea) | Tian ltane 1 4 ft tans * satalene "| Here, ATA = [ 1 wns] apt -! tanx 1 | Sec?x[tane ~2ianx 1-t 1_[t-tantx sec"x| 2tanx mADE EASY 116. 417. Linear Algebra 65 WA sec? (or) Short Method: since [A Bl = [Al [Bl () Rank of A= 1 Because each row will be scalar multiple of first row. So we will get only one non-zero row in row Echeleaon form of A, Alternative: Rank of A= 1 Because all the minors of order greater thant will be zero. Sol Given system of equations has no solution if the lines are parallel i¢., their slopes are equal 118. 119. 120. (a) 81% + Aya + Aya = 8, App + Appty + yyy = by yy + apy + Aaa a1 ae aslin] [bh 1 A A ||22| = | by ay 2 alle) [by if |Al#O then AX = B can be written as X= AB. It leads unique solutions. It]A] #0 then 2 hg# Oeach A, is non-zero. It [A] #0 then all the row (column) vectors of A are tineerly independent. Sol. RP Ry, 123: -f o-44 2 0 00: k-2 For infinite may solution (A: B) = pla) < number of variables (A: B) (c) pregye ae + y+ pz te + py + qz=0 par Qf £|. The system is At =O rpg This is a homogenous system. Such a system has non-trivial solution iff |Al=0. par So a+r pl=0 rpd| Let A= 66 424 122, 123, for GATE and ESE Prelims Phar — p*) ~ aq? - pr) + (pq ~ °) =0 P+ + 8 —3pqr=0 P= q = rsalisfies the above equation. Also if p+ q+ r= 0 then a can be transformed into one of the row as completely O's as shown below. a Pe Pe q 4 pl=0 gq ) Therefore the correct option is (c) which is P+qtr=Oopsqzr (c) Given AX=0 (Ay, ,) = O = olor 4 ~ no @ AA 4-2) (1-a)- 10 =0 -5h-6=0 A-OA+ = a= 6-1 Maximum eigen value is ‘6’ Given eigen value A= 1 Lox X be the vector, Then [4 - AX = 0 WA 1 2 0 1-2 0 |x = 1 2 1-2 puth=1 0 -1 2x 0 0 Offx, 12 Offxs xp + 2rq = 0 J|-o0 a) 42x = Kwe get x, = -KI2 and x, = 1 So the eigen vector =k] -1/2 V4 mabe EASY Linear Algebra 67 The ratios are x/x, Hence ratio of two eigen values = ae 4 } 2 So option (d)is correct. and x/r3= 2S 4 128. (b) For singular matrix ag Only option (b) (-4, 2, 1) has the same ratios and therefore is @ correct eigen vector. @) 24 ak P. 129. Lot 2, and A, be the eigen value of matrix A 4S tarps = 7 forp=2 ‘Sum of eigen value =Aythy=24p (i) Product of eigen value = Aghg= 20-1 ii) g aT = a From equation (i) = Tythy a, pre ee ed From equation (i) > BAB = 2p-1 2 =f P#Z) - 29-1 ct 4 ) p 14 > ee Cans OR 24 1p IA~l=0 130. 2k 4 Ps pale? (2-A)(p-a)-1 <0 22=(p + 2)k + (2p-1)=0 By putting values of p from options. 14 By putting option (4) = in above equations 5 ives value 5, >. gives value §, 5 lAl =0 ‘According to properties of eigen value Product of eigen values = | Al =0 = Alleast one of the eigen value is zero. (b) Characteristic equation | A~A1|=0 3-2 0 2 1-1-2 0 |=0 0 a -2-al ~B+A)[1+4)(242)-O]-2a-) = 0 2a= (+ 1)(A + 2)(043) (A+ 1) (0? + 5A +6) 2a = {0 +62? + 11h +8) 20a oh ABE + 122+ 11) =0 (foramaximaand minima) B24 1411 =0 (b) For a matrix containing complex number, eigen values are real it and only if Az AS= (Ay 10 54) 4 As|x 2 2 l4 2 ~10} 68 SEMMEMELEE for GATE and ESE Prelims MADE ERsy 10 4 136. (c) A9= (A =|5-j 20 2 alli] 42 -10 sily arty=a By comparing these, x+8y=b 5~J a= 10x? + 102+ 12xy = 1 131. (d) Ellipse with major axis along (| +a-6 ae 137. (d) 4) [i] fo) p-2 Determinant = Product of eigen values ° dl alo volte] (a- 4b) = 3} |2} [of [4 5-4b=-7 ° -ab = -12 . > 2as — 2 ass fromhere a= - ane 132. () b-3 Ais skew-symmetric 133. (c) Given that Mé= for MY = For M&K+ = 7 he ae ee +88—-fo} Mt = Meee We can represent the system of equation in mati formas 134. Sol Trace of A= 14 ale a+5424b=14 23 3iy Beer 0 [5 9 -6|lz det(A) = 100 4] ja 37 (A: B)= 5) s}o 2 4] - 100 [5 9-6: o| 00 b By elementary operation A, > R,~ (BR, ~ 5x2x ax b= 100 12-3 a |i 10.ab = 100 (a:j=|2 3 3 b ab = 10 (i) 00 0 : c-8a-b| From equation ()) and (i For consisting of system, ¢-3a~ b= 0 ad 139. (a) 135. (b) j25 2 Result, Rank (AT A) = Rank (A) AG 2] mAOE ERSY Linear Algebra 69 Fi, + 2R, 144, (d) 25 2 az[e* 0 13 26 oo. tay = 26 Trace = sum of eigen values « ya-2 25 = 0+ jv +0-j0 ar+ by=2 |A| = product of eigens 2+ 5(-2) =2 6210 = (+ jn)(o~ jo) = 0° + 0? meee which is possible only when x =~ 2-12 7 « x=6 145. Sol. 140. (0) Two eigen values are 2 + iand 3 of a3 x 3 matrix. 141 142, 143. |, m-< a (system may still be inconsistent so incorrect) Ml, m> n(rank may stil be equal to n of hence solution may exist so incorrect). IIL. rit= n(some system rank may be equal ton and hence may have solution so correct). So only Il is correct. (0) Consider the ‘2 x 2" square matrix m=|2 od = 2 (a+ d+ (ad-be)=0 (i) Putting A= 1, we get 1-(a+ d) + ad-bo=0 1-a-d+ad-(1-oft-a)=0 t-a-d+ad-1+atd-a 0 = Owhich is true. 1 salistied the eq.) but = 2,3, 4 doesnot satisty the eq, (i). For all possible values of a, ° Sol 144 A=|1 1 1/Eigen value are 0, 0, 3 14 1 (a) 24 All Eigen values of A= [i | are positive 2>0 2x2 leading minor must be greater than zero 24 feo 2k-1>0 2k>4 kot 2 146. 447. 148. The third eigen value must be 2 - 7 Now, lal lAl=(2+) (2-1 x3 24-72) x3 25x32 15 (a) i say Eigen values are’, and A, We can write wo=[" 2 o elt Response due to initial conditions, x(01= (9 -x(0) le" 0 fo] _ 0% 21) (a) Eigen values of A? ~3A + 4/ are = (1 =3(1) + 4and (-2)? -3(-2) + 4 = 2,14 Note: A°x = 12x = Xis eigen vector for 4? corresponding oeighen value A? X, and X, are eigen vector of A corresponding to 1,-2 Then X, and X, are eigen vector of A?-3 + 41 corresponding 102. 14. en Sol, Consider [A~ dul (2-AN2-23-2)= 0 4=2.2,3 70 for GATE and ESE Prelims MADE ERsy 2.=3 there is one Li. Eigen vector =2 Consider (A~2N x= 0 rank=2_ The equation are x, = 0 No. of variables = 3 % Let.x, = kbe independent. Ky ft Eigen vectors |0| = k}0 o} lo Only one independent Eigen vector in the case ofk=2 Hence finally no. of Li. Eigen vectors = 2 149. Sol. Aas an eigen value as zero JAl=0 a2 4 eee io 6 -4 -9+x 63 + 2x +52)-2(-81 + Oe +78) + 4(-36+42)=0 37x11) - 2(9r-3) + 4(6) = 0 2ix-93-18x + 6 + 24=0 Be-3 = eet 150. Sol. Eigen values of given matrix A are 1 ~1, Eigenvalues of Avare 9 ~ Eigenvalues of 3A’ are 3,327 Eigen values of A?- 34 are -2, -4, 0 trace of A°-3A?=-2-44 0=-6 181. Sol. Eigen(A) = 1, 2,49 A= 1x2x4=8 1 al = 0.125 Now, (a"t| 152. (d) By Calyey Hamilton theorem, P=2 153. Sol. 154, 156, -A+h-120 W-2-1=0 By Cayley Hamilton Theorem A-AT=0 Aa Ast Als AP42AST=AbI+2AGT =3A+2r AB = OAR + 128447 = A+ 1+ 1204 41= 21 As 13 Als AS Als 144A + 897 233. 144 “(ie All x12] _ [233 14a yf (ral Tree os fo] x[12] = 233 (o) Consicier a random matrix which satisfy propery of square symmetrix matrix contain real values O58 ie, A= whose eigen values +5 and 5 Obys -5. Since, || ie, 45-5 =0. Since, rank of matrix is 2, so atleast one eigen value would be zero for every square matrix size n> 2.Forn=2 1248 DOA Sit 25 and sum of trace |A| =0 F423 < 50 Both A, and A, are real, since Ais real symmetric matrix, which means atleast one elgen value 3 ‘would be in range [-6, §]. But since in our example ‘no eigen value greater than 5. So II" statement is wrong So, option (b) is only correct. Sol. The product of eigen value of always equal to the determinant value of the matrix 2, = 10 A, =unknown | Al = 50) Ay Ay = 50 10(A,) = 80 he mane EASY Linear Algebra TA 156. Sol. 1 2 : ‘50 70] (32) (1-2) 70 20] 1 : Eigen values of A are Ay, A (ee yon - Ry + hy = 130 dy =~ 900 (i ag +9? ia+ 4) j)=0 70 hy - 80 Given thats, = | 59 am dy 80) ait x= [70 Ae soi 775 | 159. 70 Ay ~ 5600 + 70 2, ~ 3600 70 (Ay + bq) ~ 9100 70 (130) - 9100 9100 - 9100 = 0 157. (b) Product of Eigen values = determinant value 2{3 - 6) + 1 (8-0) =2-3)+8=-6+8=2 188. (d) ind 0 160. lz ° x re {a) IPI = \° 1 ‘| 2°23 re ° al {b) For P to be orthogonal P x PT = Ayto 4 v2 || V2 E oO cee Ae 4 V2 ILV2 v2 f1 00 =|0 10 001 161. (©) Since P is orthogonal, its inverse is equal to its transpose (d) For eigen values (c) Characteristics equation is IA — All = iva -1 5 0 5-A 6] = 0 6 5-3) (1 ANG - AP + 36) = 0 (1-2) (2-104 + 61) = Aad 1 00-28 _ 404121 rr rn A= 1,5 4 6 @ Coordinate transformation matrix cos8 sind ~[Ete ced Orthogonal, @ = 90° Coordinate transformation matrix of mirror image [sind cose [es Sel _ [singor 890° -sing0' [4] (a) The characteristics equation is 1A ~ M1 joa 1 0 | 50) 0 ciee [o -3 -4-a] “MAD +1? + 3) ~ 1(0 0) = “hQ2 + 4h +3) =0 =O, A+ 1043) n for GATE and ESE Prelims mADE ERsu 162. (c) From here, ‘The given matrix is symmetric and all its eigen lA) = 4 values are distinct. Hence all its eigen vectors | are orthogonal one ofthe eigen vector isx, = |0 |A-a| =0 1 HA 2 89 4 5 The corresponding orthogonal vector in the given 5 +h 2 39 4 1 eo ien oh alto option is C. ie. x, = | 0 3 4 5 1A 2 -1 2 3 4 5 1-4 ‘Sum of all elements in any one row must be zero, ie, 15-220 k= 15 163. Sol, 165. (0) {1-10 00 5 10 10 001-10 N= A=|0 1-100 efi oa] T1000 1 Ss oo 1 ROR, Ry Ry + A, 7 el) 5 10 10 [3 6 6 5A, and R, — A, - 3A, Ay > Ry + A 2 0 100 000 Ry > Ry + Ry Which is in Echelon form _ Rank of matrix Mis, joo 1 +0 AM) = 2 Sloearaal 166. Sol ooo 4 ‘| 0-4-2 eee eae Ay © Pyand Ry > Ry + Ry la 8 8 1-10 00 IP+ Ql =-16 + 16 =0 01-400 So, rank #8 =[00 1-410 O = 0 Tako the 2x 2 minor [' ero oo +44 ls 9 00 | So, rank of P+ Qis 2. SR nnde EASY Linear Algebra 3 > 467. Sol 170. (a) 1h) = 29-92 + ah + 30=0 . Now 2 is one of roots of this equation agr-|' 1] [; q] . fe idl So, 2-4x 2+ax2+30=0 (6 2]17 4] [32 56 8-16 +2a+30=0 = a=-t 171, (ce) So, the equation is 4? - 42? ~ 112 + 30 =0 Given that P is inverse of @ Now, by polynomials division we get P= =a ar =aqt 3_ 42 MAAPAIR4H ey 4g Goan 4-2 roots of 4? ~ 2h - 15 = O are a 172 = 2tVEFO 248 5 yg ' a 7 1 So the eigen values are 2, and.-3, the maximum Ghorerap Rian absolute eigen value is 5 168. (c) 169. The equation Ar whet is given “] x = b becomes according to (2, &) 9] where a, are column vectors in A" but since we have C, (not all zero) such that, Dice #0 it means the n column vectors are nol linearly independent and hence rank (A) <9 So we have infinitely many solutions one of which will be J,, where J, denotes a n-aim vector of all 1 (a) [3 2 l= lat 3-2, [A- 2a) = [ e ml 1A-all=0 (3-2) (1-2)-8=0 3-44 422-8 =0 W-4h-5=0 5-Sh-A the ~6h4 4=3,3 Algebraic muttiptcity of eigen value 3is 2. thas only one independent eigen veotor exists. 173. (d) xtyeza4 (1 xoytz20 (2) Beeyez=5 (3) Adding (1) and (2) & (2) and (3) gives 2c-42z= dand ax + 27=5 which gives = 1,2 = tandy=2 ‘Alt’ Option (b) can be eliminated since they do ‘at satisfy Ist condition. Only (d) satisfies 3rd equation. 174, (a) Ais square matrix of order 100has 15 distinct eigen values. The of minimal polynomial is a divisor of the characteristic polynomial. Every root of minimal polynomial is also a root of characteristic equation a) 7 175. 4 74 EXERC for GATE and ESE Prelims MADE EASY [21 42 -14] 24281 4 WTx,] _ for oe 4 -2+2N8i Lx} ~ Lo 4 at 42 AGL = I-19 49 a9 242(Bix, = Ax, - oo | 4 «2 | 4 24208 | 3 8 2] )[ 4 i be [-L2a ;_AdgQ_}2 3 6 2, 2+ 2NBe ia 17 7 7 Complex Eigen values and complex Eigen 62 3 vectors 7 7 77. OF 018 othogonl eet) ‘ oe 110-2 2022 176. (c) An[P OR? Option (a): Al = 6-5 =1 Option (b): 1al = Fa-> Ry 2, Fly Ry 4R, Option (c): Al 110 Option (dl): Al = 8 - 0-22 6| Hence matrix (c) is singular. 0-33 9) 17. (6) aon 2 4 ae - 0226 1A-all =0. pas a ae Ee 0 Number of non zero rows = 2 4-2-4 rankof A = 2 4-24 Dh 4224 16=0 22412 =0 179. (b) “4d ot A= 4287 (Complex eigen values) ee a) Ae 28: vd Consider (4-An)x'=0 at fear 4 Ta} fo Ayam [4 tot lo To [4 -2-26iLe Fy ~ 4P, Fy +77, 2-2NBixy = 4x5 tot ot a 0 5 3 4 2-. nami 0 -10 6 [*I-Letel a 2-2) Ry+an, [0 8 8 (2) As -28i oo 0 Consider (A-A)x=0 Number of non zero rows = 2 rank = 2 mabe EASY Linear Algebra 75 180. Sol. A=1,2,-2 Eigenvalues of Aare Eigen valuesof A? are. Aare 4Aare Slare 181. 182, 183. 184, (c) Eigen vectors corresponding to distinct eigen values are linearly independent Hence, “S2 implies $1" Sol. AX = Ohas infinitely many solutions So |Al <0 Gr ek ke W9~ 248 + 2k? = 0 (2 ky k=0,2 = "two" distinct values of k Sol. Ais 2.x 2 mattix TA Ay tay WA) =5 AP +a2=5 Oy HARES A PHAZE DRA 16 = 542d, " ae 2 uW qe. lA z 755 Sol ft o-t Ae Ie eno lo 0 -2 \A-vl] =0 !-2 0 “1 2-a 0 0 (12) (2-2) -2-4)-100-0) = 0 A A244 + 5] are A =@aya 185. (a) 186. 187. 188. N=0 Let Eigen values of Nare A, ho, hy Eigen values of N? are 22, But W2=0 = MW =0,03=003=0 Eigen values of Nare 0, 0,0 Sol (r= 2) = 2-1 0 15 Smallest value of x is t x? Gx + (c) Put k= -3 in options, Gr + 2ky=-2 => Sx-6y Kx + 6y=2 => Sv-6y=-2 Both the equations are reducing in the same form, thus the system will have infinite number of solutions. Sol. (1-2)(2-2)-2 on, The largest eigen value is 3 76 for GATE and ESE Prelims 189. (d) eigen value is repeated since if eigen values were distinct we will get one more independent eigen vector. So, II P has repeated eigen values is true. 200 | need not be true since E al has repeated eigen values and yet itis invertible. II is true since if a 2 x 2 matrix has only one linearly independent eigen vector, surely it cannot be diagonalized. 190. (b) fi re | be eigen vector for eigen value. a P) lol be eigen vector for eigen value Ae = hx 4 ayn o alla 1+ 2a rT 191. 192. MADE ERsy () 3-1 -f A=|-13 -1 1-413 Characteristic equationis, |A-al] = 0 I-A 1-1 -1 3-’ +1 “1-1 3a] (3-AN(3~ A)? = 1) + 1(-3 + 2-1) -1(1 43-4) @-ANP- CR 9-1) 44-46 (3-4) Q? ~ 64 +8)-8 + 22 BA? - 18) + 24-23 + OR - BA- B+ 2H, (23 + 922 ~ 242 + 16 = 0) 9-912 + 20, - 16 = 0 aetad (b) Consider (AB) = Infinite many solutions. Calculus 2.1 Limit 2.1.1 Definition A number A is said to be limit of a function f(x) at x= aif for any arbitrary chosen positive integer €, however small but not zero there exist a corresponding number 8 greater than zero such that f(x) Al <€ forall values of x for which 0< | x~a| <8 where the absolute value of (x - a) without any regard to sign ~almeat 2.1.2. Right and Left Hand Limits lfxapproaches a from the right, that is, rom larger ve the right hand limit of f(x) and is written as lue ofr than a, the limit of fas defined before Lt f(s) or +O)or Lt fx) Working rule for finding right hand limit is, put a + hforsvin f(x) and make h approach zero. In short, we have, f(a+0) = Lt fla h) Simitarly fx approaches a from left, that is from smaller values of x than a, the limit of fis called the lefthand limit and is written as; Lf) or la-O)or_Lt A) In this case, we have, f(a~0) = Lt fla-f) IF both right hand and left hand fimit of f, as x > a exist and are equal in value, their common value, ‘evidently, willbe the limit of fas.x—> a. If however, either or both of these limits do not exist, the limit of fasx— a does not exist. Even if both these limits exist but are not equal in value then also the limit of fas. x» a does not exist when Lt fix) = Lt fy) then Lt f(x) = e also allowed imit of a function can be any « limit of a function, values fo 2.1.3 Various Formulae These formulae are sometimes useful while taking limits, oln=9 2, Ain Mn~2) 9 , (Tay = Tenet al

You might also like